Bar Refresher Final

Ace your homework & exams now with Quizwiz!

A state university adopted a new regulation prohibiting certain kinds of speech on campus. Students, staff, and faculty convicted by campus tribunals of violating the regulation were subject to penalties that included fines, suspensions, expulsions, and termination of employment. The regulation was widely unpopular, and there was a great deal of public anger directed toward the two tenured professors who had drafted and promoted it. The following year, the state legislature approved a severable provision in the appropriations bill for the university declaring that none of the university's funding could be used to pay the two professors, who were specifically named in the provision. In the past, the professors' salaries had always been paid from funds appropriated to the university by the legislature, and the university had no other funds that could be used to pay them. If the professors challenge the constitutionality of the appropriations provision, is the court likely to uphold the provision? (A) No, because it amounts to the imposition by the legislature of a punishment without trial. (B) No, because it is based on conduct the professors engaged in before it was enacted. (C) Yes, because the Eleventh Amendment gives the state legislature plenary power to appropriate state funds in the manner that it deems most conducive to the welfare of its people. (D) Yes, because the full faith and credit clause requires the court to enforce the provision strictly according to its terms.

(A) No, because it amounts to the imposition by the legislature of a punishment without trial.

Congress passes a law making it a crime in the District of Columbia to operate any commercial motor vehicle that is not equipped with a specified noise suppression device. If the constitutionality of the law is challenged in an appropriate proceeding, the law should be declared .... (A) valid as an exercise of the congressional police power over the District of Columbia; (B) valid as a reasonable exercise of the congressional power to protect the environment; (C) invalid under the Equal Protection Clause; (D) invalid under the Necessary and Proper Clause

(A) valid as an exercise of the congressional police power over the District of Columbia;

A state university adopted an admissions policy that set aside a portion of spaces in its incoming freshman class for the admission of racial minorities, with the percentage of open slots at the school equal to the percentage of each racial minority residing in the state. This number would be adjusted every year based upon demographic information provided by the state. A stated goal of the policy was to promote educational diversity and to redress unintentional discriminatory effects of the university's past discrimination against racial minorities. A number of white applicants who were denied admission to the university sued certain university officials to enjoin the operation of the admissions policy, contending that the policy violated their rights under the equal protection clause of the Fourteenth Amendment. Assuming proper standing, how should the court rule? (A) In favor of the applicants, because the use of a quota system is not substantially related to the important interest of remedying past discrimination in university admissions. (B) In favor of the applicants, because a school may not use a quota system to enroll minority students, even if the expressed purpose is to remedy past discrimination. (C) In favor of the university officials, because the use of a quota system is necessary to the compelling state interest of remedying past discrimination. (D) In favor of the university officials, because the quota system is substantially related to the important interest of remedying past discrimination in university admissions.

(B) In favor of the applicants, because a school may not use a quota system to enroll minority students, even if the expressed purpose is to remedy past discrimination.

Doctor, a resident of the city of Greenville in the state of Green, is a physician licensed to practice in both Green and the neighboring state of Red. Doctor finds that the most convenient place to treat her patients who need hospital care is in the publicly owned and operated Redville Municipal Hospital of the City of Redville in the state of Red, which is located just across the state line from Greenville. For many years Doctor had successfully treated her patients in that hospital. Early this year she was notified that she could no longer treat patients in the Redville hospital because she was not a resident of Red, and a newly adopted rule of Redville Municipal Hospital, which was adopted in conformance with all required procedures, stated that every physician who practices in that hospital must be a resident of Red. Which of the following constitutional provisions would be most helpful to Doctor in an action to challenge her exclusion from the Redville hospital solely on the basis of this hospital rule? (A) The bill of attainder clause. (B) The privileges and immunities clause of Article IV. (C) The due process clause of the Fourteenth Amendment. (D) The ex post facto clause.

(B) The privileges and immunities clause of Article IV.

A United States ambassador was appointed by a former President with the advice and consent of the Senate. Three days after taking office, a new President requests the ambassador's resignation after the ambassador posted a photo of himself in the nude. Upon the ambassador's refusal to resign, the President issues an order purporting to remove the ambassador from office. Is the presidential order valid? (A) Yes, because there was cause for the ambassador's removal from office; (B) Yes, because the President has the power to remove ambassadors without cause; (C) No, because the President may not remove an ambassador without the advice and consent of the Senate; (D) No, because the Senate has not ratified the order by consenting to the appointment of a new ambassador.

(B) Yes, because the President has the power to remove ambassadors without cause;

A federal statute provides benefits to workers and their families for job-related injuries. A train engineer became a quadriplegic when his train collided with a freight train and derailed. His medical expenses, including physical therapy and a home health aid, were covered by the statute. Several years later, the worker, who breathed with the aid of a ventilator, developed pneumonia and died. His wife applied for survivors' benefits for herself and the worker's children. Prior to the worker's death, the family consisted of husband, wife, two children born of the marriage, and a boy, who showed up on the family's doorstep two years before the railway accident claiming he was the worker's natural child. The worker accepted him as such, and a subsequent DNA test established paternity. The boy is now 19 years old. The survivors' benefits regulation under the statute provides that only legitimate children of the deceased worker may claim benefits. Will the boy prevail if he challenges the regulation as discriminatory? (A) Yes, because the regulation is not necessary to further a compelling state interest. (B) Yes, because the regulation is not substantially related to an important state interest. (C) No, because the federal government has plenary power to determine distribution of federal employee benefits. (D) No, because the federal government may make reasonable distinctions between legitimate and illegitimate children.

(B) Yes, because the regulation is not substantially related to an important state interest.

Congress enacted a statute prohibiting the use of mechanical power hammers on all construction projects in the U.S. Subsequently, a study conducted by a private research firm concluded that nails driven by mechanical power hammers have longer-lasting joining power than hand-driven nails. After learning about this study, the city of Green enacted an amendment to its building code requiring the use of mechanical power hammers in the construction of all buildings. This amendment to the city's building code is (A) unconstitutional, because it was enacted subsequent to the federal statute (B) unconstitutional, because it conflicts with the provisions of the federal statute (C) constitutional, because the federal statute does not expressly indicate that it supersedes inconsistent state and local laws (D) constitutional, because the long-term safety justifies some additional risk to the people engaged in the construction of the buildings

(B) unconstitutional, because it conflicts with the provisions of the federal statute

The State of Tennessee recently passed a law that reduced the speed limit on its state highways from 70 to 60 miles per hour. The state highway department had just completed a 10-year study that showed overwhelmingly that traffic accidents and fatalities were lower on state highways with a 60 mile per hour speed limit. Many of Tennessee's state highways go to the Tennessee state border and connect to roadways in adjacent states. Each of the states bordering Tennessee has a speed limit of 70 mph on its state highways. A national trucking company filed a federal lawsuit claiming that the new Tennessee speed limit violates the Dormant Commerce Clause because it forces interstate traffic to slow down when it enters Tennessee. The Tennessee speed limit is most likely: (A) Unconstitutional, because Tennessee has discriminated against out-of-state traffic for the benefit of in-state traffic. (B) Unconstitutional, because Tennessee's legitimate interest in traffic safety does not clearly exceed the speed limit's burden on interstate commerce. (C) Constitutional, because the speed limit's slight burden on interstate commerce does not clearly exceed Tennessee's legitimate interest in traffic safety. (D) Constitutional, because the safety of state highways is a purely local matter that may be regulated by the states as they please.

(C) Constitutional, because the speed limit's slight burden on interstate commerce does not clearly exceed Tennessee's legitimate interest in traffic safety.

The head of a city's Department of Public Works was a jovial and gregarious fellow. He enjoyed his job, and his workers loved working for him. Although the city hired its maintenance workers "at will," the head of the Department of Public Works made it a point to let his workers know that once they were on the job for at least seven years, he would never fire them unless they erred so badly as to threaten his job. When one of his workers reached the seven-year mark, the head of the Department would throw an afterwork party at a local Veterans of Foreign Wars hall to honor the employee. At one of these parties, the guest of honor, an employee who had reached his seventh anniversary of employment with the Department, got a bit too drunk and started a fistfight with a co-worker. The next day, the head of the Department fired the employee, saying that he couldn't bear to have disharmony in his department. The city manager denied the employee a hearing on the matter. The employee sued the city for reinstatement and back pay. How should the court rule? (A) In favor of the city, because the city's maintenance workers were hired "at will." (B) In favor of the city, because the employee did not have a property interest in his job. (C) In favor of the employee, because the refusal to give the employee a hearing violated his due process rights. (D) In favor of the employee, because the injury to the employee's reputation from being fired is a protected liberty interest.

(C) In favor of the employee, because the refusal to give the employee a hearing violated his due process rights.

A state law made it a criminal offense for any state employee to "knowingly provide educational services or extend welfare benefits" to a foreign national who was in the United States in violation of U.S. immigration laws. The principal of a public elementary school was prosecuted under the law for enrolling and providing education to several foreign nationals he knew to be in the country illegally. All of these actions took place before the new law was adopted. No federal statute applied to the principal's actions. What constitutional provision would be most helpful to the principal's defense? (A) The due process clause of the Fourteenth Amendment. (B) The equal protection clause of the Fourteenth Amendment. (C) The ex post facto clause of Article I, Section 10. (D) The privileges or immunities clause of the Fourteenth Amendment.

(C) The ex post facto clause of Article I, Section 10.

A statute regulating the practice of midwives in a state was passed at midnight at the end of the regular legislative session. There were few legislators in attendance at the time of the vote, because most of the work of the legislature had been done and most members had gone home for the holidays, although a quorum existed. The sponsor of the legislation finally succeeded in persuading a core group to vote for the bill, which included a provision that the founder of the midwives' lobbying group would no longer be licensed under the state's laws. The group's founder was a practicing midwife who had lobbied successfully against health maintenance organizations and on behalf of a number of state laws guaranteeing patients' right to treatment in the area of pregnancy and childbirth. The bill's sponsor saw the midwives' successes as having driven up the cost of health care treatment throughout the state. If the group challenges the revocation of the founder's license in federal court, the likely outcome is: (A) the state will prevail, because regulation of the group does not implicate interstate commerce. (B) the state will prevail, because a license is a privilege that can be revoked without procedural due process. (C) the lobbying group will prevail, because the revocation constitutes a bill of attainder. (D) the lobbying group will prevail, because the revocation violates members' right to work under the privileges and immunities clause.

(C) the lobbying group will prevail, because the revocation constitutes a bill of attainder.

A local environmental protection association brought a lawsuit under the Clean Water Act against an industrial manufacturer for illegally discharging a variety of pollutants into a river. One member of the group owned a lake about four miles from the manufacturing plant. His family had to reduce their use of the lake due to pollutants coming directly from the manufacturer. Another member complained that the pollutants affected his enjoyment of canoeing, fishing and swimming in his lake that stood in the path of the polluted river. He had stopped swimming and did almost no fishing in the lake anymore. The federal district court granted a dismissal of the action, holding that none of the members of the plaintiff group had shown an injury in fact, and the suit was dismissed for lack of standing. On appeal, what is the likely decision of the Circuit Court of Appeals? (a) The Court of Appeals will likely hold that the Plaintiffs have suffered an injury sufficient to meet the requirements of standing, and thus the lower court decision will be reversed. (b) The Court of Appeals will likely hold that the complaints of the association and its members is based on speculation and conjecture, and thus the lower court decision will be upheld. (c) The Court of Appeals will likely uphold the lower court's decision because the plaintiffs have not shown an economic injury. (d) The Court of Appeals will reverse the lower court opinion because all citizen complaints under the Clean Water Act must be heard if any of the plaintiffs lives within 50 miles of the plant.

(a) The Court of Appeals will likely hold that the Plaintiffs have suffered an injury sufficient to meet the requirements of standing, and thus the lower court decision will be reversed. (They have suffered a personalized injury)

The United States had long recognized the ruling faction in a foreign country as that country's government, despite an ongoing civil war. Throughout the civil war, the ruling faction controlled the majority of the country's territory, and the United States afforded diplomatic immunity to the ambassador representing the ruling faction. A newly elected President of the United States decided to recognize a rebel group as the government of the foreign country and notified the ambassador from the ruling faction that she must leave the United States within 10 days. The ambassador filed an action in federal district court for a declaration that the ruling faction was the true government of the foreign country and for an injunction against enforcement of the President's order that she leave the United States. The United States has moved to dismiss the action. If the court dismisses the action, what will be the most likely reason? (a) the action involves a nonjusticiable political question; (b) the action is not ripe; (c) the action is within the original jurisdiction of the U.S. Supreme Court; (d) the ambassador does not have standing

(a) the action involves a nonjusticiable political question;

The state requires persons applying for state welfare assistance, driver's licenses, admission to the state university, or certain other state benefits to list their social security numbers as part of their applications. State agencies refer to an applicant's social security number as his or her "Central File Number." The plaintiff brings an action in federal court against certain state officials for an order enjoining them from using social security numbers in this manner. In support of her position, the plaintiff argues that at some time almost all citizens of the state apply for some form of state benefit, and the compilation of a central file on each citizen of the state is likely to have a chilling effect on the exercise of the rights granted by the First Amendment to the United States Constitution. Clearest reason for dismissal of the PS suit is that: (a) the action is unripe; (b) the question presented is moot; (c) under the 11th Amendment, state officials are immune from suit; (d) the case presents a political question

(a) the action is unripe;

A state enacts the Continuing Professional Education Act (CPEA), which provides that all persons licensed by the state to practice any profession other than medicine are required to complete 10 units per year of state-approved continuing education studies as a condition of renewal of their professional licenses. The day after the statute goes into effect, a law school graduate, who has applied for but not yet received a license to practice law, sues in federal court seeking a declaratory judgment that the CPEA is unconstitutional. Which of the following is the clearest ground for dismissal of this action by the court? (a) no substantial federal question is presented; (b) the suit presents a non-justiciable political controversy; (c) the student lacks standing to attack the statute; (d) the validity of the statute has not yet been determined by a state court.

(c) the student lacks standing to attack the statute;

(1) A defendant is on trial for a string of murders. The prosecution calls the psychiatrist who examined the defendant to testify that the defendant's psychological profile is consistent with that of a serial killer. Is the psychiatrist's testimony admissible? (A) No, because the probative value of the evidence is substantially outweighed by the danger of unfair prejudice. (B) No, because the psychiatrist's testimony is not probative as to the issue of the defendant's guilt. (C) Yes, because the psychiatrist's testimony is based on facts perceived before the hearing. (D) Yes, because the evidence tends to make the defendant's involvement in the murders more probable than it would be without the evidence.

A

(18) A small town police officer pulled over a driver for speeding. He believed that the driver was acting irritable and fidgety, but he had no articulable reason to think anything was wrong. He searched the car anyway and found two cartons of freshly canned peaches, which were owned by the driver's neighbor and reported stolen off of her porch 24 hours earlier. Authorities charged him with theft under the state criminal code. His motion to suppress the evidence because of an unlawful search was denied. On appeal, will the appellate court likely reverse the lower court decision denying the motion to suppress? A) Yes, because the search was unconstitutional due to the officer having no probable cause that would justify searching the car. (B) Yes, because when a car is pulled over for speeding, the officer must always obtain a search warrant prior to making any search. (C) No, the stop and the search were within the normal bounds of propriety for a speeding stop. (D) No, because driver being fidgety is enough for a probable cause full search of the vehicle.

A

A carpenter from State A enters into a contract with a company, in which the carpenter is to be paid $40,000 to construct customized wood shelving units throughout the company's home office and principal place of business, located in State B. The carpenter completes the work, but the company fails to pay the carpenter as required by the contract. A plumber from State C enters a contract with the same company. The plumber, pursuant to the terms of the contract, is to be paid $50,000 to update the plumbing throughout the company's home office and principal place of business The plumber completes the work, but the company fails to pay the plumber as required by the contract. May the carpenter and the plumber aggregate their claims together and sue the company in federal court? (a) No, the carpenter and the plumber may not aggregate their individual claims against the company, regardless of how similar the claims are; (b) No, because the Federal Rules do not provide for aggregation claims; (c) yes, because the claims are sufficiently related to permit aggregation; (d) Yes, because aggregation is necessary to meet the amount in controversy requirement.

A

A carpenter was recently transferred from his job in the city to his company's woodshop in the suburbs. He did not have the opportunity to visit the suburbs prior to moving there, so he bought a new home through a realtor without seeing the site. When the carpenter finally arrived at his new home, he was shocked to find that it was next door to a home designed to house and rehabilitate drug addicts. The carpenter checked all local zoning laws, but found that there was no basis for challenging the location of the rehabilitation house. Instead, the carpenter constructed a large sign in his front yard that read, "Get lost, drug addicts" in large letters. A recovering drug addict who was living in the rehabilitation house saw the sign and became very upset and intimidated. As a result of the sign, the drug addict suffered a relapse of his addiction and was unable to leave the house for several weeks. If the drug addict sues the carpenter for false imprisonment, what is the likely outcome? (A) The drug addict will not prevail, because the carpenter did not intend that the drug addict be confined to the rehabilitation house. (B) The drug addict will not prevail, because he was not restrained from leaving the rehabilitation house. (C) The drug addict will prevail, because the carpenter knew that recovering drug addicts were residents of the rehabilitation house. (D) The drug addict will prevail, because the sign in the carpenter's front yard could be interpreted as a threat.

A

A city has a public park that is filled with monuments. One of those monuments is associated with the Christian religion. The city receives a letter from a member of the religion of Qudonar. The letter provides that adherents of the religion of Qudonar have solicited funds for the creation of a monument that depicts the Eight Aphorisms of Qudonar. The city rejects the application, saying that it does not have sufficient room for the monument. Several members of the religion of Qudonar sue the city, claiming that the city has engaged in unconstitutional viewpoint discrimination. What is the likely result? (A) For the city, because monuments in a public park are a form of government speech; (B) For the city, because the monument would create the perception that the city is endorsing the religious precepts of Qudonar; (C)For the challengers, because the city has engaged in viewpoint discrimination by having a Christian-based monument but rejecting the Qudonar-based monument; (D) For the challengers, because the city leaders were biased in their views on Qudonar.

A

A construction worker sued an insulation manufacturer in federal court, claiming that he had developed a chronic health condition as a result of 20 years of exposure to the manufacturer's insulation at his work sites. The manufacturer answered, denying all liability and stating that it had never supplied its insulation to the worker's employer. The worker's attorney deposed the manufacturer's president, and the manufacturer's attorney deposed the worker. Immediately thereafter, the manufacturer moved for summary judgment on the ground that the worker had no evidence showing that the insulation had ever been used by the worker's employee. What would be the worker's best response to the motion for summary judgment? (a) Argue that more time is needed for additional discovery to show the manufacturer's liability, and attach a declaration describing the desired discovery; (b) Argue that the motion should be denied, because a central issue in the case will be the manufacturer's credibility on the question of its distribution of the insulation, and only a jury can decide questions of credibility; (c) Argue that the motion will be denied, because the manufacturer failed to attach any evidence to its motion to show that the insulation was not used by the worker's employer. (d) Make a cross-motion for summary judgment arguing that the manufacturer has introduced no evidence to show that its insulation did not harm the worker.

A

A contractor agrees to remodel a homeowner's house. During the remodeling, one of the contractor's employees drops a hammer, which strikes and injures the homeowner's child. The homeowner sues the contractor in federal court on a negligence theory. The case proceeds to trial. In the interests of what the judge believes to be justice, the judge permits a jury of 13 to decide the case. The jury returns a verdict in favor of the homeowner. Was the court's decision regarding the jury proper? (a) No, because a jury may only consist of 12 jurors. (b) No, because a jury must consist of a minimum of 6 and a maximum of 12 jurors. (c) Yes, because a jury must consist of a minimum of 4 and a maximum of 16 jurors. (d) Yes, because a jury must consist of at least 12 jurors.

A

A contractor from State A enters into a contract with a homeowner from State B, in which the contractor agrees to build a guesthouse for the homeowner. The contractor fails to complete the project as required by the contract, and the homeowner files a complaint in federal court. However, the contractor does not have an attorney. The contractor calls his cousin Vinny, who is studying to be a lawyer, for advice. Vinny tells the contractor that it is necessary for a copy of the complaint and summons to be served on the homeowner. The contractor asks Vinny how service of process is accomplished, and Vinny says, "That's simple. All you need to do is personally take the complaint and the summons to the homeowner yourself. That's called personal service." Based on this advice, the contractor personally takes the summons and complaint to the homeowner and delivers the document into the homeowner's hands. Was the homeowner properly served with the complaint and summons? (a) No, because the contractor personally served the complaint and summons; (b) No, because the contractor should have merely taped the documents to the homeowner's front door; (c) Yes, because the plaintiff must serve the defendant with the complaint and summons; (d) Yes, because a defendant in a lawsuit must receive service of process by delivery of the documents in person.

A

A defendant has pleaded not guilty to a federal bank robbery charge. The principal issue at trial is the identity of the robber. The prosecutor has called the defendant's wife to testify to the clothing that the defendant wore as he left their house on the day the bank was robbed, expecting her description to match that of eyewitnesses to the robbery. Both the defendant and his wife have objected to her testifying against the defendant. Should the wife be required to testify? (A) No, because the wife has a privilege not to testify against her husband in a criminal case; (B) No, because the defendant has a privilege to prevent his wife from testifying against him in a criminal case; (C) Yes, because the wife's viewing of the defendant's clothing was not a confidential communication; (D) Yes, because the spousal testimonial privilege does not apply in criminal cases

A

A defendant was charged with manslaughter. At the preliminary hearing, the magistrate dismissed the charge on the grounds that the evidence was insufficient. The prosecutor then brought the case before a grand jury. After hearing the evidence presented by the prosecutor, the grand jury refused to return an indictment. The prosecutor waited a few months until a new grand jury had been impaneled and brought the case before that grand jury, which returned an indictment charging the defendant with manslaughter. The defendant moves to dismiss the indictment on double jeopardy grounds. Should the motion be granted? (a) No, because jeopardy had not attached; (b) no, because there has been no conviction or acquittal; (C) yes, because any proceeding after the preliminary hearing would violate double jeopardy; (d)yes, because bringing the case before the grand jury was a violation of double jeopardy

A

A driver and a trucker are involved in an accident on a state road. The driver sues the trucker in federal court, alleging that the trucker's negligence caused the accident. The driver seeks $80,000 in damages. The case proceeds to a jury trial, during which the court gives an erroneous jury instruction. The jury returns a verdict in favor of the trucker. The driver appeals, and specifically, the driver raises the issue regarding the court's erroneous charge to the jury. The appellate court finds that the erroneous charge of the jury would not likely have affected the result of the trial. What rule did the appellate court apply when deciding the issue regarding the erroneous jury instruction? (a) Harmless error rule; (b) Minimal error rule; (c) Non-prejudicial rule; (d) Substantive error rule

A

A driver from State A was operating a 1963 Corvette Stingray sports car on a highway in State B and was involved in a crash with a motorcyclist from State B. The motorcyclist was miraculously unhurt in the crash. The driver believed that the accident was caused by the motorcyclist's recklessness, and the driver filed a negligence claim in federal court in State B, seeking $75,000 from the motorcyclist for the damage sustained to the driver's car. Does the federal court have original jurisdiction over the case? (a) No, because although the opposing parties are citizens of different states, the amount in controversy is $75,000. (b) No, because negligence is not a case or controversy arising under federal law. (c) Yes, because the Commerce Clause gives federal courts original jurisdiction over negligence claims that arise from accidents occurring on highways in the United States. (d) Yes, because the opposing parties are citizens of different states, and the amount in controversy is $75,000.

A

A driver was badly injured when her car ran off the road for unknown reasons and hit a tree, and the car's airbags did not inflate. The driver sued the manufacturer for product liability, alleging that there was a defect in the airbags' inflating mechanism. At the jury trial, the plaintiff's lawyer offered testimony by two other consumers that the airbags in their cars, which were the same model and manufacturer as the driver's car, also failed to trigger during collisions. The manufacturer's lawyer objected to the evidence, arguing that the speed and other circumstances in the two witnesses' collisions had not been shown to be substantially similar to those of the present case, creating a large risk that these witnesses' testimony would mislead the jury and be unfairly prejudicial to the defense. The trial judge allowed the two witnesses to testify, and the jury returned a verdict in favor of the plaintiff. The manufacturer appealed; one of its arguments on appeals was that the two witnesses' testimony about their airbags' failure should have been excluded as irrelevant, misleading, and highly prejudicial. What standard will the appeals court in evaluating this argument? (a) an abuse of discretion standard; (b) a clearly erroneous standard, applied while giving due regard to the trial court's opportunity to judge the proposed witness' credibility; C) a de novo standard, giving no special deference to the trial court's decision to admit the evidence; (d) a harmless error standard, requiring reversal unless the reviewing court believes that the evidence ruling was not only erroneous but probably changed the trial's outcome.

A

A high school student's teacher told her that she was going to receive a failing grade in history, which would prevent her from graduating. Furious, she reported to the principal that the teacher had fondled her, and the teacher was fired. A year later, still unable to find employment because of the scandal, the teacher committed suicide. The student, remorseful, confessed that her accusation was false. If the student is charged with manslaughter, her best defense would be ... (a) she committed no act that proximately caused the teacher's death; (b) she did not intend to cause the teacher's death; (c) she did not act with malice; (d) she acted under extreme emotional distress

A

A motorcyclist is traveling at a high rate of speed on a highway when a deer runs into the road. The motorcyclist swerves to avoid the deer and loses control of the motorcycle. The motorcyclist crashes into a bicyclist who is injured in the crash. The bicyclist files a claim based on negligence in federal court against the motorcyclist. Before the parties hold an initial conference to plan discovery, the bicyclist serves the motorcyclist with a set of interrogatories. Was it proper for the bicyclist to serve the motorcyclist with a set of interrogatories? (a) No, because the parties had not yet held an annual conference; (b) No, because interrogatories may not be served until depositions are taken; (c) Yes, because no initial conference is necessary before engaging in discovery; (d) Yes, because the bicyclist is the plaintiff in the case.

A

A painter, who is driving a van, is involved in an accident with a driver. The painter files a lawsuit based on negligence against the driver in federal court. During the proceedings, the court enters a ruling that does not permit the admission of a piece of evidence offered by the painter. While the case is pending, the painter desires to appeal the court's ruling, and the painter asks his attorney to do so. May the attorney appeal the court's ruling? (a) No, because the court has not yet entered a final judgment in the case; (b) No, because the attorney must not permit the painter to make the legal decisions in the case. (c) Yes, because the court's ruling is adverse to the painter's interests. (d) Yes, because any order entered by a court is appealable once te order is rendered

A

A soap manufacturer makes shipments to his customers every three to six months. Realizing he had not heard from a top customer in nearly a year, the president of the company sent the customer a letter stating: "Based on our records of sales and your last shipment date, you are nearly out of your supply of our products. In order to restock your supplies for the next several months, we will send you seven truckloads of our products at the rate of $3,000 per truck, with the first delivery to be made within two to three weeks." The letter was signed by the president. The customer did not respond. Two weeks later, the company sent the first two truckloads of its products to the customer, who quickly sold the soap products to several merchants in her area. Three months later, the president sent truckloads three and four of soap products to the customer. If truckloads three and four contained goods that substantially conformed to the customer's usual order, what are the customer's legal obligations to the manufacturer? (A) The customer is obligated to pay the contract price of $3,000 each for the first four truckloads, though she may reject any of the remaining future three truckloads if the products are substantially nonconforming. (B) The customer must pay $3,000 each for the first two truckloads of soap products, but she may reject the shipment of truckloads numbers three and four and revoke the contract before any more goods are sent. (C) The customer is obligated to pay $3,000 each for the first two truckloads, but she may refuse delivery of all remaining product that the company delivers. (D) The customer must send back truckloads numbers three and four with a note to the company explaining that she was rejecting those truckloads and did not wish to receive any more of its products in the future.

A

A state department of corrections has a long-term policy of segregating prisoners by race in double cells for 30-60 days each time they enter a new state prison facility in order to classify each prisoner and determine whether they are members of a racial gang. Prison officials believe that such temporary segregation significantly reduces violence caused by gangs. An inmate who has been intermittently double-celled based on his race under the policy terms ever since his 1990 incarceration files a suit in an appropriate federal district court alleging that the segregation policy violated his right to equal protection of the laws under the Fourteenth Amendment. Who has the burden of production and persuasion on the prisoner's equal protection claim? (a) The state must prove its racial policy is justified by compelling reasons and is narrowly tailored to serve that interest; (b) The state must prove that its racial policy serves an important governmental interest and no reasonable alternatives are available by the state; (c)The state must prove the racial policy is reasonably related to a legitimate penological interest; (d) The inmate must show that the government has a compelling governmental interest that is rationally related to its policy.

A

A taxi driver from State B entered into a written agreement with a painter from State A in which the painter agreed to paint the taxi driver's home. The painter completed the work, but the taxi driver failed to pay the painter as required by the contract. Two months after the taxi driver's failure to pay the painter for his work, the painter and the taxi driver were involved in an accident on a highway in State B. The accident was unrelated to the taxi driver's failure to pay the painter. The painter sued the taxi driver in federal court on a claim of negligence regarding the accident and a claim of breach of contract regarding the painting of the taxi driver's home. The painter sought $60,000 in damages for the negligence claim and $25,000 in damages for the breach of contract claim. The taxi driver moved to dismiss the case for lack of subject-matter jurisdiction. Should the federal court grant the taxi driver's motion? (a) No, because the parties are diverse and the amount in controversy is satisfied; (b) No, because the accident occurred in the taxi driver's state of residence, and therefore the federal court has subject matter jurisdiction over the claim; (c) Yes, because the breach of contract claim is unrelated to the negligence claim; (d) Yes, because at least one claim must independently satisfy the amount of controversy requirement.

A

A wife decided to kill her husband because she was tired of his infidelity. She managed to obtain some cyanide, a deadly poison. One evening, she poured wine laced with the cyanide into a glass, handed it to her husband, and proposed a loving toast. The husband was so pleased with the toast that he set the glass of wine down on a table, grabbed his wife, and kissed her passionately. After the kiss, the wife changed her mind about killing her husband. She hid the glass of wine behind a lamp on the table, planning to leave it for the maid to clean up. The husband did not drink the wine. The maid found the glass of wine cleaning the next day. Rather than throw the wine away, the maid drank it. Shortly thereafter, she fell into a coma and died from cyanide poisoning. In a common law jurisdiction, of what crime(s), if any, could the wife be found guilty? (a) attempted murder of the husband and murder or manslaughter of the maid; (b) only attempted murder of the husband; (c) only murder or manslaughter of the maid; (d) no crime.

A

An airline passenger nearly killed in a crash is suing the airline for personal injuries. To prove the extent of his injuries, the passenger offers a videotape taken by a local news station immediately after the crash that shows serious burns covering much of the passenger's face. The airline moves to exclude the videotape on grounds that its probative value is substantially outweighed by the danger of unfair prejudice? In making this ruling, which of the following is NOT appropriate for the judge to consider? (A) the videotape will make it more likely that the passenger will win the suit; (B) there are other methods of proving the passenger's damages; (C) the videotape can be restricted to its proper purpose by instructing the jury to disregard any possible emotional appeal; (D) the videotape will encourage the jury to decide the suit on an emotional basis

A

Dan entered the police station and announced that he wanted to confess to a murder. The police advised Dan of the Miranda warnings, and Dan signed a written waiver. Dan described the murder in detail and pinpointed the location where a murder victim had been found a few weeks before. Later, a court-appointed psychiatrist determined that Dan was suffering from a serious mental illness that interfered with his ability to make rational choices and to understand his rights and that the psychosis had induced his confession. Dan's confession is .... (a) admissible, because there was no coercive police conduct in obtaining Dan's consent; (b) admissible, because Dan was not in custody; (c) inadmissible, because Dan's confession was a product of his mental illness and was therefore involuntary; (d) inadmissible, because under these circumstances, there was no valid waiver of Miranda rights

A

Plaintiff heads a company that sells photography equipment. Defendant, an individual who owns a film school, bought 500 cameras from Plaintiff. Defendant received the cameras but refused to pay, insisting that the goods did not meet industry standards. Plaintiff sued Defendant in federal district court for breach of contract, invoking the court's diversity jurisdiction. Plaintiff knows from publicly available sources that Defendant has purchased studio lighting from a particular lighting company. Plaintiff wants to find out, through discovery, whether Defendant also defaulted on that contract. Which statement best describes whether Plaintiff may get discovery about any contract between Defendant and the lighting company? (a) Plaintiff may, during a deposition of Defendant, ask about the lighting contract so long as the information is not privileged, is relevant to a claim or defense in the case, and is proportional to the needs of the case. (b) Plaintiff may not, during a deposition of Defendant, ask about the lighting contract if the answer would not be admissible at trial. (c) Plaintiff may propound interrogatories to the lighting company asking about the company's contracts with Defendant, if that information would be relevant to a claim or defense in the case. (d) Plaintiff is entitled to receive a copy of the lighting contract as a part of automatic initial disclosure from Defendant.

A

State A is a popular tourist destination. On three recent occasions, tourists have been robbed at gunpoint at isolated highway rest areas. In the most recent incident, the robber shot and killed one of his victims. On each occasion, a white sedan with a dent in the passenger side door has been sighted in the vicinity. After the robbery, which ended in murder, a trucker who saw the sedan speeding away was able to give the police a license number. The police were able to track the license number and identify a carpenter as the owner of the car. An ATM security camera also captured the carpenter's image as he attempted to use an ATM card belonging to one of his victims. After obtaining an arrest warrant, police officers went to the carpenter's home. The carpenter lived with his parents. When the officers pulled up in front of the house, the white sedan was sitting in the driveway. The officers could see the carpenter through the front window as they walked up to the front door. The carpenter, who was in the living room watching television with his parents, answered the door, and the officers arrested him. After the carpenter was handcuffed, one of the officers proceeded to walk through the house, looking in all the rooms. Finally, he went into the garage, where in plain view he saw a gun, several wallets, and several cameras. At the carpenter's trial on robbery and felony murder charges, the prosecution seeks to offer the items found in the garage into evidence. The carpenter moves to suppress this evidence on Fourth Amendment grounds. Should the carpenter's motion be granted or denied? (A) Granted, because the police did not have a reasonable suspicion an armed accomplice was present in the house and, therefore, had no right to search the house. (B) Granted, unless the police had probable cause to believe that evidence of the robberies would be found in the house. (C) Denied, because the police officers were permitted to conduct a protective sweep of the house incident to the lawful arrest. (D) Denied, because the carpenter's parents owned the house, so he had no reasonable expectation of privacy in the place searched.

A

State A is a popular tourist destination. On three recent occasions, tourists have been robbed at gunpoint at isolated highway rest areas. In the most recent incident, the robber shot and killed one of his victims. On each occasion, a white sedan with a dent in the passenger side door has been sighted in the vicinity. After the robbery, which ended in murder, a trucker who saw the sedan speeding away was able to give the police a license number. The police were able to track the license number and identify a carpenter as the owner of the car. An ATM security camera also captured the carpenter's image as he attempted to use an ATM card belonging to one of his victims. After obtaining an arrest warrant, police officers went to the carpenter's home. The carpenter lived with his parents. When the officers pulled up in front of the house, the white sedan was sitting in the driveway. The officers could see the carpenter through the front window as they walked up to the front door. The carpenter, who was in the living room watching television with his parents, answered the door, and the officers arrested him. After the carpenter was handcuffed, one of the officers proceeded to walk through the house, looking in all the rooms. Finally, he went into the garage, where in plain view he saw a gun, several wallets, and several cameras. At the carpenter's trial on robbery and felony murder charges, the prosecution seeks to offer the items found in the garage into evidence. The carpenter moves to suppress this evidence on Fourth Amendment grounds. Should the carpenter's motion be granted or denied? (A) Granted, because the police did not have a reasonable suspicion an armed accomplice was present in the house and, therefore, had no right to search the house. (B) Granted, unless the police had probable cause to believe that evidence of the robberies would be found in the house. (C) Denied, because the police officers were permitted to conduct a protective sweep of the house incident to the lawful arrest. (D) Denied, because the carpenter's parents owned the house, so he had no reasonable expectation of privacy in the place searched.

A

The state constitution in one state guaranteed the right to marry to same-sex and opposite-sex couples alike. A political group got a proposition on the ballot, and a majority of the people voted to eliminate the right of same-sex couples to marry. The proposition did not affect any of the other rights that same-sex couples may have possessed from existing state law. Persons affected sued the state government for unconstitutionally taking away the rights that they had and doing it for no legitimate purpose, in violation of the due process clause of the Fourteenth Amendment. Will the court grant the requests of the lawsuit and declare the proposition void and reinstate the rights of same-sex couples to marry? (a) Yes, under the due process clause no compelling state interest justifies denying same-sex couples the fundamental right to marry. (b) Yes, because under the Fourth Amendment it would be an illegal interference against the right of privacy guaranteed to everyone. (c) No, because the people voted to take a provision out of the existing constitution and when that is done by public vote it cannot be altered. (d) No, because the ballot proposition was properly placed on the ballot and all statutory procedures were followed, thus precluding the courts from taking adverse action.

A

Three persons are being considered to serve on a jury: Juan, Mary, and Bill. Juan is a citizen of Mexico, and he is 18 years of age, fluent in English, and does not have a criminal record. Mary is a citizen of the United States, and she is 17 years of age, fluent in English, and does not have a criminal record. Bill is a United States citizen, and he is 57 years of age, fluent in English, and he has a criminal record of two felony convictions. Which of the following is a correct statement regarding who may serve on a jury? (a) Juan, Mary, and Bill are all disqualified from serving on a jury. (b) Juan, Mary, and Bill may all serve on a jury. (c) Juan is disqualified from serving on a jury, but both Mary and Bill may serve on a jury. (d) Juan and Mary are disqualified from serving on a jury, but Bill may serve on a jury.

A

An ordinance of the city of Green requires that its mayor must have been continuously a resident of the city for at least five years at the time he or she takes office. Candidate, who is thinking about running for mayor in an election that will take place next year, will have been a resident of Green for only four and one-half years at the time the mayor elected then takes office. Before he decides whether to run for the position of mayor, Candidate wants to know whether he could lawfully assume that position if he were elected. As a result, Candidate files suit in the local federal district court for a declaratory judgment that the Green five-year-residence requirement is unconstitutional and that he is entitled to a place on his political party's primary election ballot for mayor. He names the chairman of his political party as the sole defendant but does not join any election official. The chairman responds by joining Candidate in requesting the court to declare the Green residence requirement invalid. In this case, the court should: A. refuse to determine the merits of this suit, because there is no case or controversy. B. refuse to issue such a declaratory judgment, because an issue of this kind involving only a local election does not present a substantial federal constitutional question. C. issue the declaratory judgment, because a residency requirement of this type is a denial of the equal protection of the laws. D. issue the declaratory judgment, because the Candidate will have substantially complied with the residency requirement.

A. refuse to determine the merits of this suit, because there is no case or controversy. (would be an advisory opinion)

A man and his friend were watching a televised football game at the man's home. Upset by a penalty called by the referee, the friend threw a bottle of beer at the man's television, breaking the screen. Enraged, the man picked up a nearby hammer and hit the friend on the head with it. The friend died from the blow. The crimes below are listed in descending order of seriousness. In a jurisdiction that follows common law principles, what is the most serious crime of which the man could properly be convicted? (a) murder (b) voluntary manslaughter (c) involuntary manslaughter (d) assault

A; He intended to inflict serious bodily harm by picking pup the hammer.

A collector of old musical instruments acquired a 250-year-old violin, which he offered for $45,000 to an old friend of his, who was a retired performer. The collector made the offer on February 15. The retired performer, who hadn't performed in a long time, wanted some time to think about the purchase. Meanwhile, an aggressive investment specialist from New York had heard of the collector's acquisition of the violin and wanted to acquire it as an investment. The investment specialist called the collector daily, trying to get the collector to set a price. The collector decided to keep the collector's offer to the retired performer open until February 25 in exchange for a payment of $100, which the retired performer made. At the same time, the aggressive investment specialist insisted on paying $50,000 for the violin and insisted on tendering the purchase price to the collector. On the morning of February 25, the retired performer decided she would buy the violin. The retired performer called the collector first thing in the morning, but there was no answer at the collector's apartment. The retired performer went to her bank, got a cashier's check for $45,000, and sent it to the collector as she was leaving town to conduct a violin symposium in Europe. Who owns the violin? (A) The investment specialist owns it, since $100 would not be adequate consideration on an option contract for such an expensive purchase. (B) The investment specialist owns it, if the collector sold it to the investment specialist on February 26 before receiving the retired performer's check. (C) The collector owns it, since a writing is necessary to enforce any contract worth $500 or more. (D) The retired performer owns it, since the retired performer had effectively accepted the collector's offer and had a valid option contract with him.

B

A consumer was driving a car late at night when the headlights went out, causing the consumer to run off the road and hit a tree, injuring him badly. The consumer brought a product liability suit in diversity against the car manufacturer, a large publicly-traded corporation. The suit alleged that the headlight system failed on account of a defective design. The consumer demanded a jury trial. During voir dire, one prospective juror testified that he owned $100 of publicly-traded stock in the defendant manufacturer. Under questioning by the judge, the prospective juror said that she was confident that her stock ownership would not interfere with her ability to be impartial in the case. The consumer's lawyer has challenged the prospective juror for cause. Should the judge dismiss the juror for cause? (a) No, because the juror's financial stake is sufficiently small; (b) Yes, because there is a presumption that any direct financial relationship between a juror and a party will impair the juror's ability to be impartial; (c) Yes, so long as there is at least one other potential juror in the pool who would not be challengeble for cause; (d) No, so long as the judge believes the juror will be impartial

B

A customer from State A files a product liability suit in federal court in State C against a corporation that is incorporated in State A. The corporation directs, controls, and coordinates its activities from State A. The customer seeks $100,000 in damages. The corporation does not have any contacts with State C, but believing that it would be in the interest of fair play and justice to defend the lawsuit in State C, the corporation consents to the federal court having subject-matter jurisdiction over the case. Does the federal court have original jurisdiction over the case? (a) No, because subject-matter jurisdiction is not proper in a products liability case. (b) No, because subject-matter jurisdiction cannot be consented to or waived by the parties. (c) Yes, because subject-matter jurisdiction can be consented to when the consent is given based on interests of fair play and justice. (d) Yes, because diversity of citizenship exists between the federal court and the parties, and the amount in controversy is satisfied.

B

A man entered a casino, pointed a gun at the cashier, demanded money and got away with over $10,000. The man has been arrested and charged with the armed robbery of the casino. At trial, the prosecution offers evidence that when the man was arrested, the arresting officer found a small amount of marijuana in the man's shirt pocket. Is the evidence of the marijuana admissible? (A) No, because such character evidence may be offered only to rebut evidence of good character after it is offered by the defendant. (B) No, because its probative value is substantially outweighed by the danger of unfair prejudice. (C) Yes, because such evidence tends to prove that the man is predisposed to commit crimes. (D) Yes, because the prosecution may establish the man's general bad character.

B

A man walks down the street and notices two members of the Army. The man is a vehement, anti-war activist and genuinely believes that the United States is a militaristic country with dangerous foreign policy initiatives. When the two Army members approach, the man nearly chest bumps one of the members of the Armed forces, points at his own t-shirt which reads "**** the Military" and curses at the two men. A melee ensues, in which all three parties are injured. The police come to the scene and arrest the man for violating a disorderly conduct ordinance that prohibits individuals from engaging in "violent or fighting behavior or conduct." If the man challenges his charges with a First Amendment defense, what is the most likely result? (A) for the government, because the man uttered a true threat at the military men; (B) for the government, because the man engaged in fighting words; (C) for the man because he engaged in offensive political speech that did not rise to the level of fighting words; (D) For the man because members of the military, like the police, are held to a higher standard.

B

A man's car broke down, and since he was only a mile from the repair shop, he decided to walk. As he approached the repair shop, he tripped on the cracked front cement step. The man fell and fractured his wrist. A woman who was jogging by rushed up to him and asked, "Are you hurt?" The man said, "Yes." The jogger then remarked, "It is unbelievable that a business would allow its front step to deteriorate to this state." A few months later, the man filed an action against the repair shop to recover for his injuries. At trial, the jogger's statement is (A) admissible, even though it is hearsay. (B) admissible, as an excited utterance. (C) inadmissible, as hearsay not within any recognized exception. (D) inadmissible, if the declarant cannot be subpoenaed to testify at trial.

B

A motorcyclist is involved in an accident with a driver. The motorcyclist files a lawsuit based on negligence against the driver in federal court. The case proceeds to a jury trial. During the trial, the driver offers a particular piece of evidence. The court permits the introduction of the evidence over the motorcyclist's objection. Eventually, the jury enters a verdict in favor of the driver. The motorcyclist appeals the judgment, and in particular, the motorcyclist alleges that the trial court erred in permitting the introduction of the evidence over the motorcyclist's objection What standard should the appellate court apply when viewing the motorcyclist's appellate claim? (a) de novo (b) abuse of discretion (c) clearly erroneous (d) reasonable doubt

B

A motorcyclist is involved in an accident with a driver. The motorcyclist sues the driver in federal court, alleging that the driver's negligence caused the accident and $80,000 in damages. After making three challenges for cause regarding potential jurors, without seeking the court's permission, the motorcyclist's lawyer challenges a fourth potential juror for cause on the ground that the juror is the driver's brother-in-law. The driver objects to the challenge on the ground that the motorcyclist has already used the total number of challenges for cause permitted by the Rules. Will the driver's objection be successful? (a) No, because a party may make up to five challenges for cause without permission from the court. (b) No, because a party may make an unlimited number of challenges for cause on grounds that a juror has a bias or relationship to a party. (c) Yes, because a party may only make three challenges for cause on grounds that a juror has a bias or relationship to a party. (d) Yes, because a party may only make one challenge for cause without permission from the court.

B

A patient from State A sues a doctor from State B in state court in State B. The patient's complaint seeks $100,000 in damages for the doctor's alleged negligence. On the advice of the patient's attorney, the patient decides that the suit will have a better chance of success in federal court in State A. Twenty days after service of the initial complaint on the doctor, the patient files a notice of removal containing a short and plain statement of the complaint of the grounds for removal, together with a copy of all process, pleadings, and orders served upon the defendant. Is removal to the federal court in State A proper? (a) No, because notice of removal was filed more than 10 days after the doctor had been served with the initial complaint; (b) No, because only defendants may remove cases to federal court; (c) Yes, because the notice of removal was timely filed; (d) Yes, because the notice of removal was timely filed, and the federal court has diversity jurisdiction over the case.

B

A patient sues a doctor in federal court on a claim of medical malpractice. Both parties are represented by attorneys. The parties engage in discovery, and the case is on track to proceed to trial. As close to the trial date as possible, the court holds a final pre-trial conference. The patient, the patient's attorney, and the doctor attend the conference. Did all of the required parties attend the conference? (a) No, because the patient's attorney attended, and the purpose of the final pre-trial conference is for the parties, not the attorneys, to have one final opportunity to settle the matter; (b) No, because the doctor's attorney was absent; (c) Yes, because at least one representative from each side attended the conference; (d) Yes, because attendance and participation in a final pre-trial conference is optional.

B

A police department began to receive reports of violent attacks that typically occurred in the afternoon, just after the local high school let out for the day. The police put together a task force utilizing informants and surveillance of after-school hangouts to gather intelligence concerning students who might have gang affiliations or connections. Once the police compiled this list of gang-associated students, they asked the principal of the high school to summon the listed students to the office and search them, and to contact the police if any potential evidence was found. Of the 25 students searched, only one was found to be carrying a weapon. In the student's ensuing trial, his lawyer moves to suppress the weapon. What is the likely outcome of the student's motion? (A) The motion will be granted, because the police did not conduct the search. (B) The motion will be granted, because there was neither probable cause nor a search warrant in support of the search. (C) The motion will be denied, because the student had no reasonable expectation of privacy in school. (D) The motion will be denied, because the school did not need a warrant to search a student.

B

A police officer stopped a driver who had run a red light. Upon approaching the car, the officer noticed a strong odor of alcohol and immediately asked whether the driver had been drinking. The driver admitted having had several alcoholic drinks that evening. The driver, charged with a DUI, moved to suppress the officer's testimony regarding the driver's statement about his drinking. The driver argued that the officer had elicited the statement without providing the requisite Miranda warnings. The prosecutor has responded that the statement should be allowed in the prosecution's case-in-chief or, at a minimum, should be allowed as impeachment in the event the driver testifies and denies drinking. How should the court rule regarding the driver's statement admitting his drinking? (a) the statement should be allowed, because although the driver was in custody, the officer's spontaneous utterance upon smelling alcohol did not rise to the level of interrogation; (b) the statement should be allowed, because the driver was not in custody for Miranda purposes when the statement was made; (c) the statement should be suppressed both in the prosecution's case-in-chief and as impeachment evidence, if the driver testifies; (d) the statement should be suppressed in the prosecution's case-in-chief, but it may be used as impeachment evidence if the driver testifies.

B

A real estate developer offered to hire an architect to design a massive memorial. A week later, before the architect had accepted the offer, the developer told the architect he had changed his mind. Instead, the architect learned, the developer had decided to hire his own daughter. Does the architect have any legal recourse? (A) No, because adequate consideration was never discussed. (B) No, because the architect had not yet accepted the contract. (C) Yes, because the developer's act violated the spirit of fair dealing in the common law. (D) Yes, because one week is a legally unreasonable amount of time to revoke an offer

B

A retailer brought a federal diversity action against a wholesaler, alleging breach of contract and fraudulent misrepresentation. After the parties presented their evidence at trial, the court instructed the jury on the law. Neither party filed a motion for judgment as a matter of law before the case went to the jury. The jury found for the retailer on both claims. After the court entered judgment on the verdict, the wholesaler moved for a new trial and for judgment as a matter of law, arguing that the evidence was insufficient to support the jury verdict on either claim. The court acknowledged that there had been problems with some of the evidence, but it denied the motion. The wholesaler appealed, challenging the sufficiency of the evidence. Should the appellate court consider the wholesaler's challenge? (a) no, because a determination of the sufficiency of the evidence is solely within the jury's province. (b) No, because the wholesaler did not raise the sufficiency of the evidence issue in a motion for judgment as a matter of law before the case went to a jury. C) Yes, because the challenge was raised and ruled on by the trial court before the wholesaler filed the appeal. (d) Yes, because, as the trial court acknowledged, the wholesaler has strong arguments on the challenge.

B

A sophisticated businessman from State A desired to open a franchise of a restaurant chain owned by a corporation, and he filed an application for the franchise in the corporation's district office in State A. In order to open a franchise, the businessman completed several training sessions sponsored by the corporation, including a week-long conference provided at the corporation's headquarters in State B. Eventually, the businessman entered into a contract with the corporation. The contract permitted the businessman to open a franchise in State A. The contract further provided that the parties' relationship was established in State B and that State B's law governed the contractual relationship. After opening the franchise, the businessman struggled to make a profit due to an unexpected downturn in State A's economy, and he was unable to meet his contractual obligations to the corporation. The corporation filed a breach of contract lawsuit in State B against the businessman. The businessman objected to personal jurisdiction in State B. Will the businessman's challenge to the court's jurisdiction be successful? (a) No, because the contract provided that State B's law governed the contractual relationship, which is sufficient to establish minimum contacts with State B; (b) No, because the businessman was sophisticated, and he engaged in a substantial and continuous business relationship with the corporation, and he received fair notice that he could be subject to personal jurisdiction in State B. (c) Yes, because the businessman operates the franchise in State A, and State A is where the corporation's district office is located. (d) Yes, because the failure to meet his contractual obligation was due to an unexpected decline in the economy.

B

A web site developer offered to pay an aspiring model $100 to use the model's photo on his web site. The model replied, "Let me think about it." Assuming that the model would ultimately accept the offer, the developer posted the photo on his web site. Several weeks later, the model informed the web site developer that she did not wish to accept his offer. Which of the following statements is most accurate? (A) No contract existed between the web site developer and the model because the developer never tendered the model the $100. (B) No contract existed between the web site developer and the model because the model never accepted the offer. (C) A contract existed between the web site developer and the model because the model's failure to reject the offer in a timely manner constituted an implied acceptance of the offer. (D) A contract existed between the web site developer and the model because the model's failure to object in a timely manner to the developer's use of her picture constituted an implied acceptance of the offer

B

An inventor from State A holds a patent for a device that increases fuel economy in gasoline-fueled combustion engines. An opportunist from State B learns of the inventor's device and begins manufacturing and selling an identical device, marketing it as the opportunist's own invention. The inventor sues the opportunist in federal court in State B. The inventor's complaint sought $500,000 in damages and alleged patent infringement under federal law. The opportunist denied the allegations by filing an answer to the complaint. The day after filing the answer, the opportunist moved to dismiss the action for lack of subject-matter jurisdiction. Should the court grant the motion to dismiss? (a) No, because the inventor's claim satisfies the amount in controversy? (b) No, because the inventor's claim arises under federal law. (c) Yes, because the inventor's claim does not arise under federal law. (d) Yes, because although the inventor's claim arises under federal law, the opportunist denied the allegations by filing an answer to the complaint.

B

An investor is currently on trial for the sexual assault of an elderly woman. The investor's girlfriend is called to the stand by the prosecution for the purpose of incriminating the investor. During cross-examination, the investor's attorney asks the girlfriend, "Isn't it true that you have also been charged with being an accessory to the sexual assault that your boyfriend is currently being tried for?" The girlfriend answers in the affirmative. The investor's attorney then asks, "And isn't it true that the prosecutor agreed to drop those charges in exchange for testimony that would incriminate your boyfriend?" The prosecution objects. Should the objection be sustained? (A) No, because the girlfriend's answer would constitute an admission. (B) No, because the girlfriend's answer would constitute proper impeachment evidence. (C) Yes, because in order to impeach a witness in this manner, there must be proof of a conviction. (D) Yes, because it is against public policy to admit evidence that is essentially an offer to compromise.

B

An online seller from State A sells various products and owns and operates a website that is accessible worldwide. The online seller routinely sells products through the website to residents of State B, and a large portion of those sales are generated by marketing e-mails sent by the seller to residents of State B. The e-mails contain a link to the website. The website permits potential customers to view images of the items for sale, and customers can enter their contact, address, and credit card information on the website. A resident of State B accesses the online seller's website in State B and purchases a product. The product malfunctions, and the resident is injured. The resident sues the manufacturer of the product, as well as the online seller, in federal court in State B. The online seller challenges the court's personal jurisdiction. Will the challenge to the court's personal jurisdiction be successful? (a) No, because the online seller owns and operates the website; (b) No, because the website is not passive, and the online retailer seeks business with State B's residents; (c) Yes, because the online seller is from State A and is not incorporated in State B; (d) Yes, because the resident of State B did not access the website while being physically present in State A.

B

As a driver is backing his garbage truck out of an alley, he slams into a passing car. Several bystanders call police and emergency services on their mobile phones to report the accident. When police arrive, they take extensive reports and document the names and current addresses of all the people present at the accident scene, regardless of whether the people claim to have witnessed the accident. The driver of the passing car, who was injured by the garbage truck, sues both the driver and the city for her injuries and the property damage to her car. At trial, she calls one of the bystanders to testify that just as the garbage truck was backing out of the alley, a worker on the back of the garbage truck started jumping up and down, waving and yelling, "No, no, stop! It isn't clear! You can't go now!" Is this testimony admissible? (A) Yes, but only for purpose of impeachment, if the garbage truck driver offered conflicting testimony on the stand. (B) Yes, because it is an excited utterance. (C) No, because it hearsay not within an exception. (D) No, because the coworker is available to testify to the circumstances.

B

On January 1, the student sent his original menu and recipes to Healthy Life magazine for entry in the competition. He simultaneously submitted copies to the Head Chef's office. On April 15, Healthy Life magazine notified the student that he had won the Healthy Chef contest. The student was delighted, and in reliance upon the promised one-year scholarship, he did not submit his tuition payment for the following year. When he was contacted by the school's bursar's office regarding the unpaid tuition, he told the administrator that his tuition was covered by a one-year scholarship. The administrator informed the student that if he did not submit tuition payment within 15 days, he would be expelled. Is the Head Chef's one-year scholarship offer enforceable? (A) Yes, because the school's unilateral contract became a binding bilateral contract when the student notified the dean's office of his intent to enter the competition. (B) Yes, because the cooking school's notice of conditions to enter the cooking contest constituted a unilateral contract, which the student accepted by entering and winning the contest. (C) No, because the scholarship offer was contingent upon the fulfillment of a condition involving Healthy Life magazine, an outside party. (D) No, because it was not certain that the student would win the contest.

B

The owner of a small boat repair business wanted to move into the business of restoring luxury yachts. One day the owner passed by the shop of a competitor who restored luxury yachts and had two yachts stored in dry-dock in his business yard by the harbor. The owner wanted to restore both yachts. The owner also knew that he would have to make a different financing arrangement with his bank in order for them to carry him while he got into the yacht repair business. The owner proposed to the competitor that the competitor sell the owner an option on the two yachts. The competitor agreed and wrote out a note offering to sell the owner the larger yacht for $250,000 and the smaller one for $175,000. The owner offered the competitor $10,000, and the competitor agreed to keep the offer open for 15 days from May 1, the current date. The owner left a check with the competitor and left with the note. On May 7, the owner sent the competitor a note indicating the bank had told the owner that the bank felt the price of $250,000 for the large yacht was too high. The owner also indicated that the bank was only considering granting him a loan of $225,000 for the large yacht. The owner also indicated that the deal would be off unless the competitor would come down in price on the large yacht. What the owner did not know was that the competitor had already sold the large yacht for $260,000 on May 8. Later, on May 11, the owner sent the competitor another note indicating that the bank had further reviewed the owner's first quarter business figures and had decided that the owner was now a better candidate for the loan. The owner indicated in the May 11 note to the competitor that the owner would buy the large yacht for $250,000 and the small one for $175,000. If the competitor had sold the large yacht on May 8, after receiving the owner's note, and the owner became aware of this the same day, the owner's power to accept the offer (A) terminated on the day he learned of the sale of the large yacht. (B) terminated when the competitor sold the large yacht. (C) terminated when the competitor received the note on May 7. (D) continued until May 15, because the owner had purchased an irrevocable option.

B

The plaintiff filed a complaint in federal district court and served it on the defendant. The defendant filed and served an answer 15 days later. Another 15 days later, the defendant filed and served an amended answer. Was the amendment of the answer proper? a) Yes, because an answer can be amended at any time. (b) Yes, because an answer can be amended once within 21 days after serving it. (c) No, because any amendments had to be filed within 21 days of service of the complaint. (d) No, because an answer cannot be amended once it is filed

B

The plaintiff filed a complaint in federal district court and served it on the defendant. The defendant filed and served an answer. Twenty-five days later, the plaintiff's attorney called the defendant's attorney and asked for his permission to file an amended complaint. The defendant's attorney orally agreed to allow the plaintiff to file an amended complaint. The plaintiff filed an amended complaint. Did the plaintiff properly amend his complaint? (a) Yes, because the adverse party agreed to allow the amendment. (b) No, because he failed to obtain either leave of court or written consent of the adverse party. (c) Yes, he amended the complaint within 30 days of service of process. (d) No, because he had to file any amendment within 21 days after service of process.

B

Wanda is an employee of the National Park Service. The Park Service recently created a new personnel level for field employees - Senior Ranger III, which is the highest salaried position available to Park Service field employees. The position is restricted to employees over six feet in height. Wanda seeks your advice as to whether she can challenge the validity of the height restriction in federal court. If you decide to file suit on her behalf (she is only 5-3), which of the following would be your best argument against the validity of the restriction? (a) Since most women are less than six feet tall, the restriction is unconstitutional under the Equal Rights Amendment; (b) Since most women are less than six feet tall, the restriction is an invalid discrimination on the basis of gender in violation of the Due Process Clause of the Fifth Amendment. (c) Since most women are less than six feet tall, the restriction is an invalid gender-based discrimination in violation of the Equal Protection Clause of the Fourteenth Amendment. (d) The restriction denies Wanda a property right without an opportunity for a hearing before a neutral decisionmaker, in violation of the Due Process Clause of the Fifth Amendment.

B

Congress wishes to enact legislation prohibiting discrimination in the sale or rental of housing on the basis of the sexual orientation of the potential purchaser or renter. Congress wishes this statute to apply to all public and private vendors and lessors of residential property in this country, with a few narrowly drawn exceptions. The most credible argument for congressional authority to enact such a statute would be based upon the: A. General welfare clause of article 1, section 8 because of the conduct the statute prohibits could reasonably be deemed to be harmful to the national interest. B. commerce clause of Article I, section 8, because, in the aggregate, the sale or rental of almost all housing in this country could reasonably be deemed to have a substantial effect on interstate commerce. C. enforcement clause of the Thirteenth Amendment, because that amendment clearly prohibits discrimination against the class of persons protected by this statute. D. enforcement clause of the Fourteenth Amendment, because that amendment prohibits all public and private actors from engaging in irrational discrimination.

B. commerce clause of Article I, section 8, because, in the aggregate, the sale or rental of almost all housing in this country could reasonably be deemed to have a substantial effect on interstate commerce.

One evening, a man asked his cousin if he could borrow the cousin's car to go to a job interview the following day. The location of the interview made driving preferable to taking the bus. The cousin agreed to lend the man his car. The next morning, the man picked up the car from the cousin's house and headed to the interview. However, on the way to the interview, the man decided to stop by a liquor store for a little "fortification." He bought a large bottle of vodka and drank some of its contents in the car as he traveled the remainder of the way to his interview. While the car was parked in the parking lot in front of his prospective employer's place of business, the car was struck by a large van. The driver of the van immediately drove away from the scene and was not located or identified. The collision caused $1,600 damage to the cousin's car. If the cousin sues the man for the damage to his car, the cousin will most likely recover: (A) the cost of repairing the cousin's car, because the car was damaged while under the man's control. (B) nothing, unless the damage to the cousin's car was caused by negligence on the part of the man. (C) the value of the cousin's car before it was damaged, because the car was damaged while under the man's control. (D) the value of the cousin's car before it was damaged, because the man exceeded the authorized use of the car.

B; Trying to lead down the path of showing the guy was irresponsible. But, this does not matter. Car was parked and struck while parked, not while he was driving. So, he was not negligent for purposes of that accident

A woman believes that a local high school is engaged in racially discriminatory practices in the disciplining of students. She decides that her best course of action is to picket peacefully outside of the school. She figures that many parents will view her signs and perhaps call the superintendent's office and complain about the issue. School officials contact the police, who come out to the school. The police inform the woman that she must desist from her protesting activities, because of a local ordinance,. The ordinance prohibits picketing within 300 feet of a public school. However, the ordinance has an exemption for labor picketers who are picketing peacefully pursuant to labor conditions at the school. If the woman files a First Amendment lawsuit against the government, what is the most likely result? (A) For the woman because she is engaging in protest activities on a traditional public forum; (B) For the woman because the ordinance is fatally flawed because it discriminates on the basis of content; (C) For the government because the school has a valid interest in limiting picketing near the entrances where there will be a lot of drivers; (D) For the government because there are ample, alternative ways for the woman to convey her beliefs.

B; discriminates on the viewpoint of her picketing

A state law provided free public education for all children in the state between the ages of five and 19, and made schooling compulsory for children between the ages of five and 16. Another state law provided that principals of public schools had the authority to suspend any student for up to 10 days for misconduct and that the parents of the student had the right to appeal the principal's decision to the state Board of Education. Several students at a public high school organized a walkout to protest the state's business ties with a foreign dictatorship. The principal suspended all of the members of a student group that the principal suspected had organized the walkout for 10 days based on a school rule that prohibited organized walkouts. The members of the student group requested an opportunity to plead their case to the principal, but the principal denied the request. The parents of the students suspended for 10 days sued the school district, contending that the suspension law was unconstitutional. How should the court rule? (A) In favor of the school district, because under the circumstances, the students did not have the right to a trial-type hearing. (B) In favor of the school district, because the ability to appeal the suspension to the state Board of Education was adequate procedural due process. (C) In favor of the parents of the suspended students, because adequate procedural due process mandates at least a minimum form of a hearing about the imposition of a significant disciplinary suspension. (D) In favor of the parents of the suspended students, because the students had a right to a trial-type hearing before the imposition of the suspension.

B; they have the ability to appeal the decision, and under the facts that is adequate due process.

A cyclist sued a defendant corporation for injuries sustained when she was hit by a truck owned by the defendant and driven by its employee, who was making deliveries for the defendant. The day after the accident, the employee visited the cyclist in the hospital and said, "I'm sorry for what I did." At trial, the employee testified that he had exercised due care. Why is the cyclist's testimony relating what the defendant's employee said at the hospital admissible to prove negligence? (A) It is a prior inconsistent statement; (B) It is a statement against interest; (C) It is a statement by a party-opponent's agent; (D) It is a statement of then-existing state of mind

C

A defendant was charged with possession of marijuana with intent to distribute. On direct examination, the defendant testified that he worked with disadvantaged children as a drug counselor, that he hated drugs, that he would "never possess or distribute drugs," and that he had never used drugs and would not touch them. The government then offered as a rebuttal witness a police officer who would testify that, three years earlier, he saw the defendant buy cocaine from a street dealer. The defendant objected. Is the testimony of the police officer about the prior drug transaction admissible to impeach the defendant? (A) No, because the bad act of buying drugs is not sufficiently probative of a witness' character for truthfulness; (B) No, because it is contraction on a collateral matter; (C) Yes, because it is a proper impeachment; (D) Yes, because the bad act shows a disregard for the law and makes it less likely that the defendant would respect the oath of truthfulness

C

A dentist has been arrested and charged with the robbery of a grocery store. At trial, the dentist testifies that he had nothing to do with the robbery, and was visiting his now-deceased mother at the time of the robbery. On cross-examination of the dentist, the prosecution asks the dentist, "Have you ever been convicted of perjury in the past eight months?" Will the court allow the question? (A) No, because misrepresentation has nothing to do with robbery. (B) No, because the probative value of the evidence is outweighed by the danger of prejudice. (C) Yes, because a perjury conviction can be used to show that the dentist has the propensity to lie. (D) Yes, because a perjury conviction can be used to prove that the dentist has a general bad character.

C

A driver is involved in an accident with a motorcyclist. The motorcyclist files a claim against the driver in federal court alleging that the driver's negligence caused the accident. However, before the driver serves the answer to the complaint, the motorcyclist files a notice of dismissal. Was it permissible for the motorcyclist to file the notice? (a) No, because the driver did not file a motion for summary judgment; (b) No, because a dismissal may only be obtained by a court order; (c) Yes, because the notice of dismissal was filed before the driver served an answer; (d) Yes, because the plaintiff is master of the complaint and may at any time voluntarily dismiss the case.

C

A husband and wife loved to travel. While the wife enjoyed shopping for gifts for the grandchildren, the husband visited the local nurseries. He owned a greenhouse, and he constantly sought new varieties of plants to add to his stock. While touring the California coast, the husband decided to investigate a nursery that supposedly carried an excellent selection of succulents. During his perusal of a patio area, the husband slipped in a puddle of water, which apparently dripped from an overhanging wisteria. The husband alleged he sustained severe back injuries, and he brought suit against the nursery owner. Although the nursery owner admitted the husband was injured by slipping in a puddle, the nursery owner denied negligence and claimed that the husband was contributorily negligent. At trial, the nursery owner calls one of his employees to testify that just before the husband fell, the employee heard someone call out, "Be careful, there is a puddle of water on the floor." The employee's testimony is (A) inadmissible, because it was not an excited utterance. (B) inadmissible, as hearsay not within any exception. (C) admissible, because it is relevant to the husband's contributory negligence. (D) admissible, as a statement of the declarant's present state of mind.

C

A husband and wife were both professors at the United States Naval Academy. The husband made a speech criticizing United States foreign policy with respect to a Middle Eastern country and was dismissed soon thereafter. Six months later, he accepted new employment in another state. The man's wife has commenced suit in federal court contending that the Naval Academy violated her husband's right to due process and his right to free speech when it fired him. What is the most likely ground on which the court will dismiss the suit? (a) The political question doctrine, because foreign policy is in the domain of the executive branch; (b) The case is moot, because the husband has a new job; (c) The plaintiff has no standing; (d) There is no federal question presented.

C

A man intensely disliked his neighbors. One night, intending to frighten them, he spray-painted their house with racial epithets and threats to kill them. The man was arrested and prosecuted under a state law providing that "any person who threatens violence against another person with the intent to cause that person to fear for his or her life or safety may be imprisoned for up to five years." In defense, the man claimed that he did not intend to kill his neighbors, but only to scare them so that they would move away. Can the man constitutionally be convicted under this law? (a) No, because he was only communicating his views and had not commenced any overt action against the neighbors. (b) Yes, because he was engaged in trespass when he painted the words on his neighbors' house. (c)Yes, because his communication was a threat by which he intended to intimidate his neighbors. (d)Yes, because his communication was racially motivated and thus violated the protections of the Thirteenth Amendment.

C

A mechanic told several of his friends that an engineer had battered four different women. The engineer learned of the mechanic's comments and sued the mechanic for defamation. At trial, the engineer testified that he knew that the mechanic had told several people that he abused women. The mechanic sought to introduce evidence that the engineer had battered four women. Is this evidence admissible? (A) No, because it is not proper evidence of the engineer's character. (B) No, because the danger of unfair prejudice outweighs the probative value. (C) Yes, because it is substantive evidence. (D) Yes, but only as impeachment evidence.

C

A patient filed a medical malpractice action against a hospital in federal court, alleging that hospital staff had failed to diagnose the patient's cancer based on an X-ray that had been taken at the hospital. The patient's cancer was diagnosed six months later, based on the same X-ray, when the patient sought a second opinion. In the interim, the cancer had spread. Fact and expert discovery have been completed in the action. The hospital has moved for summary judgment. In support of its motion, the hospital has submitted a memorandum identifying the facts that it claims are not in dispute. It has also cited and attached supporting exhibits, including a report from the hospital's radiologist, who found no signs of cancer on the X-ray. What is the best way for the patient to raise a genuine issue of material fact? (a) Submit a report from the patient's expert radiologist contradicting the findings in the report of the hospital's radiologist; (b) Submit an affidavit from the patient's attorney detailing his conversations with the patient's expert radiologist; (c) Submit an affidavit from the patient's expert radiologist with findings that contradict the report of the hospital's radiologist. (d) Submit the patient's medical records showing the plaintiff's current cancer diagnosis.

C

A plaintiff files a complaint alleging that the defendant failed to meet contractual obligations. The defendant's attorney files a baseless counterclaim in bad faith. The plaintiff properly files a motion for sanctions against the defendant. After hearing the matter, the court imposes sanctions against the defendant's lawyer and law firm. No other attorneys from the lawyer's firm appeared in court or filed motions in the case. Was it proper for the court to impose sanctions on the law firm? (a) No, because a court may sanction a lawyer, but not the lawyer's law firm. (b) No, because no other lawyer from the firm appeared in court or filed motions in the case. (c) Yes, because a court may sanction the lawyer and the lawyer's law firm. (d) Yes, because the plaintiff filed a motion for sanctions, which is the only way for a defendant to be sanctioned by a court.

C

A plaintiff sued a defendant in federal court for injuries arising out of an accident involving the parties. The plaintiff alleged and presented evidence at trial demonstrating that her injuries had left her legs permanently paralyzed. The jury found in favor of the plaintiff and awarded her $5 million in damages. Two months after the court entered judgment, the defendant was given a videotape made that day showing the plaintiff jogging with her doctor. What is the best way for the defendant to seek relief from the judgment? (a) Move for a new trial or in the alternative for remittitur to reduce the award in light of the shortened duration of the plaintiff's injuries; (b) Move for relief from the judgment on the ground that the judgment was based on the jury's mistaken belief that the plaintiff's injuries would be permanent; C) Move for relief from the judgment on the ground that the plaintiff committed a fraud in obtained damages for permanent injuries; (d) Move for relief from the judgment on the ground that there is newly discovered evidence that the plaintiff's injuries were not permanent.

C

A police officer notices a man and woman yelling at each other on a public street. The woman keeps yelling at the officer. The officer approaches the pair and asks them if there is any problem. He does not identify himself as an officer initially. He does identify himself as an officer, but does not threaten the woman with arrest. The woman begins berating the officer, calling him a "no good S.O.B." and "an *******." The officer then instructs her that she needs to conduct herself in a better fashion or he will arrest her for disorderly conduct/breach of the peace. The woman then begins another tirade at the officer, spewing curse words at a voluminous rate. The officer arrests the woman under a local ordinance, which provides: "Anyone who uses opprobrious or offensive language in a public place toward a public official is guilty of breach of the peace." As attorney for the woman, what is your best constitutional-based defense to the breach-of-peace charges? (A) The woman did not utter a true threat and engaged in protected speech; (B) The officer should have identified himself initially before engaging in the woman in conversation; (C) The ordinance is unconstitutionally overbroad; (D) Fighting words are a category of speech that does not ever include modern-day curses at police officers.

C

A police officer sees a robber point a pistol at a couple and take items from them before fleeing on foot. The officer chases the robber on foot. The robber, with gun still in hand, runs into a nearby apartment. This apartment is being leased by the robber. The officer follows the robber into the apartment and the officer realizes that there is no rear exit. The officer begins to search the apartment in an attempt to locate the robber and finds a large quantity of drugs when he opens a hallway closet. The robber is located under a bed in another room. The officer arrests the robber for the armed robbery and possession of drugs with the intent to distribute. At a pre-trial motion, the robber's attorney argues that the drugs are inadmissible evidence because the officer did not have a warrant to search the property. How will the court rule? (A) Inadmissible, the officer needed a warrant to enter the apartment. (B) Inadmissible, the drugs were not in plain view. (C) Admissible, the officer was in hot pursuit. (D) Admissible, there were exigent circumstances.

C

A police officer sees a robber point a pistol at a couple and take items from them before fleeing on foot. The officer chases the robber on foot. The robber, with gun still in hand, runs into a nearby apartment. This apartment is being leased by the robber. The officer follows the robber into the apartment and the officer realizes that there is no rear exit. The officer begins to search the apartment in an attempt to locate the robber and finds a large quantity of drugs when he opens a hallway closet. The robber is located under a bed in another room. The officer arrests the robber for the armed robbery and possession of drugs with the intent to distribute. At a pre-trial motion, the robber's attorney argues that the drugs are inadmissible evidence because the officer did not have a warrant to search the property. How will the court rule? (A) Inadmissible, the officer needed a warrant to enter the apartment. (B) Inadmissible, the drugs were not in plain view. (C) Admissible, the officer was in hot pursuit. (D) Admissible, there were exigent circumstances.

C

A police officer works on a multi-jurisdictional drug free task force. The officer, who is a sergeant, works very hard to secure search warrants on two stash houses that are part of a highly successful drug crew. However, when warrants are served on the premises, the two stash houses are completely empty. Suspecting that there is a leak on the task force, the sergeant goes and informs his superior officer of his concerns. Shortly thereafter, the sergeant is removed from the task force and does not receive a promotion to which he felt he was entitled. He believes that these adverse employment actions were in retaliation for his blowing the whistle on possible corruption. If the sergeant files a First Amendment retaliation claim against the police department and the city, what is the likely result? (a) for the sergeant because his speech about a possible corruption was clearly speech on a matter of public concern; (b) for the sergeant because he engaged in a form of political speech; (c) for the defendants, because the sergeant engaged in official, job-duty speech; (d) for the defendants, because the sergeant only spoke to his superior officer and, thus, engaged in more private speech.

C

A produce distributor contracted to provide a grocer with eight crates of lettuce at the distributor's listed price. The distributor's shipping clerk mistakenly shipped only seven crates to the grocer. The grocer accepted delivery of the seven crates but immediately notified the distributor that the delivery did not conform to the contract. The distributor's listed price for seven crates of lettuce was 7/8 of its listed price for eight crates. The distributor shipped no more lettuce to the grocer, and the grocer has not yet paid for any of the lettuce. How much, if anything, is the distributor entitled to collect from the grocer? (A) nothing, because the tender of all eight crates was a condition precedent to the grocer's duty to pay; (B) the reasonable value of the seven crates of lettuce, minus the grocer's damages, if any, for the distributor's failure to deliver the full order. (C) the listed price for the seven crates of lettuce, minus the grocer's damages, if any, for the distributor's failure to deliver the full order. (D) the listed price for the seven crates of lettuce.

C

A public school has a bulletin board that it uses to promulgate various school policies and events. Most of the bulletin board contains messages that have been specifically approved of by the principal's or vice principal's office. However, there is a section of the bulletin board that has been used by different faculty members to identify community events, programs, or concerts that those members think should be viewed by the school community. Mr. Patterson, a teacher in the school, posts a message on the bulletin board about an upcoming meeting of the Federalist Society, a group that generally meets to discuss significant public issues usually from a conservative position. The vice-principal, who is a die-hard member of the Progressive wing of the Democratic Party, informed Mr. Patterson that his bulletin posting needed to be removed because it was educationally unsuitable. If Mr. Patterson challenges this action, what is his best argument? (A) The bulletin board is a traditional public forum and the vice-principal committed content discrimination; (B) The bulletin board is a non-public forum but the vice-principal acted arbitrarily and unreasonably; (C) The bulletin board is a designated public forum and the vice-principal committed viewpoint discrimination by targeting Mr. Patterson's speech; (D) The vice-principal acted out of personal interest and the posting by Mr. Patterson was pure political speech.

C

A state enacts the Continuing Professional Education Act, which provides that all persons licensed by the state to practice any profession other than medicine are required to complete 10 units per year of state-approved continuing education studies as a condition for renewal of their professional licenses. The day after the statute goes into effect, a law school graduate, who has applied for but not yet received a license to practice law, sues in federal court seeking a declaratory judgment that the Continuing Professional Education Act is unconstitutional. Which of the following is the clearest ground for dismissal of this action by the court? a) No substantial federal question is presented. (b) The suit presents a non-justiciable political question. (c) The student lacks standing to attach the statute. (d) The validity of the statute has not yet been determined by a state court.

C

A teacher was driving home one night when he was stopped by a police officer for having a malfunctioning rear taillight. Acting on a hunch and based on many years of police experience, the officer asked the teacher if he could search the vehicle. When the teacher agreed, the officer founded a loaded pistol under the driver's seat. The teacher was then arrested. The teacher was unaware that the pistol was under the seat, because it had been placed there by his roommate when he borrowed the vehicle the day before. The teacher had previously been convicted of robbery and had served a term in state prison as a consequence. The district attorney charges the teacher with being an ex-convict in possession of a firearm, a statutory felony, and the teacher moves to suppress the pistol on Fourth Amendment grounds. Which of the following is the state's strongest argument in favor of permitting admission of the pistol? (A) The search was reasonable under the circumstances, because it was based upon the experience and knowledge of a veteran police officer. (B) The search was incident to a valid arrest. (C) The teacher consented to the search. (D) The search was reasonable due to the inherent mobility of a motor vehicle.

C

A trucker is traveling at a high rate of speed over icy roads. The trucker loses control of the truck and is involved in an accident with a driver. The driver files a claim based on negligence in federal court. The complaint alleges that the trucker negligently operated the truck and caused $100,000 in damages. Ten days after the filing of the last pleading directed to the issue of whether the trucker's negligence caused the damages, the trucker files a jury trial demand. Was the demand for a jury trial timely? (a) No, because the trucker did not file the demand before answering the complaint. (b) No, because the jury trial demand was not filed within 7 days of the filing of the last pleading directed to the issue for which the basis of the right to a jury trial exists. (c) Yes, because the jury trial demand was filed within 14 days of the filing of the last pleading directed to the issue for which the basis of the right to a jury trial exists. (d) Yes, because the jury trial demand was filed within 30 days of the filing of the last pleading directed to the issue for which the basis of the right to a jury trial exists.

C

A wealthy woman and her friend belonged to an exclusive tennis club. To improve her tennis game, the woman purchased a machine that would "fire" tennis balls for her to hit. She purchased a Digital Ace machine, one of the most expensive, elaborate models on the market. The friend also decided to purchase a tennis ball firing machine, but she bought a very basic model, that had fewer features and was much less expensive than the machine the woman had purchased. Over time, the friend found that she greatly enjoyed using the machine. The woman, however, began using her machine less and less and in fact had grown bored with tennis altogether. One Saturday afternoon, the woman mentioned to her friend that she was losing interest in tennis and wanted to sell her ball firing machine. The woman described the many features of the Digital Ace machine to the friend, and the friend became somewhat interested in purchasing it. One week later, on October 1, the woman mailed the friend a letter which stated that she was willing to sell the friend her Digital Ace ball firing machine, which was worth approximately $4,000, for $1,500. The letter further stated that the friend could pick up the machine at the woman's home any weeknight evening. After receiving the letter, the friend spent several weeks debating whether to replace her machine with the Digital Ace. Finally, she decided to purchase the machine from the woman. On October 14, the friend stopped in to play tennis at the tennis club. While she was there, she gave the receptionist a letter addressed to the woman. In the letter, the friend stated that she agreed to purchase the woman's Digital Ace machine for $1,500 and that she would pick up the machine at the woman's house on Monday, October 31, at 7:30 p.m. The receptionist promised to give the letter to the woman the next time that the woman came into the club. However, since the woman had stopped going to the tennis club, she never saw the friend's letter. On October 31, the woman had her annual physical, and her doctor informed her that she needed to lose 50 pounds and get back on a regular exercise program, or she might need to undergo heart surgery. Alarmed at the doctor's warning, the woman vowed that she would begin using the tennis machine every day. On the evening of October 31, the friend showed up at the woman's house to pick up the Digital Ace machine and tender payment of $1,500. The woman refused to accept the friend's check or to allow the friend to take the machine. Was the friend's letter a valid acceptance of the designer's offer to sell the Digital Ace machine? (A) Yes, because under the mailbox rule, a letter accepting an offer is effective upon dispatch. (B) Yes, because the letter contained the essential terms of the agreement and was signed by the friend. (C) No, because the woman never received the letter. (D) No, because the woman's offer could be accepted only by performance.

C

An attorney decides to increase his business by advertising. He knows that he has been running in the red for the last several months. He advertises that is an "experienced litigator." In reality, the attorney has had only one jury trial. He has settled numerous cases to his client's satisfaction A fellow lawyer complains to the Board of Professional Responsibility, who institutes proceedings against the lawyer. The attorney claims his speech is protected by the First Amendment. What is the likely result? (A) For the attorney, because attorney advertising is protected by the First Amendment; (B) For the attorney, because puffery is commonplace in nearly all forms of advertising; (C)For the Board, because his speech was false and misleading; (D) For the Board, because it has a substantial interest in regulating the speech of attorneys and because the Board sincerely believes the attorney is a danger to the public.

C

An inventor has developed a new type of plastic device that he believes may have great potential in the automotive industry. The inventor patents the invention and begins soliciting for sales of the device to various automakers. Meanwhile, an entrepreneur begins manufacturing a similar device and enters into a contractual relationship with several automakers, including ones the inventor solicited in regard to his invention. The inventor believes the entrepreneur has wrongfully infringed on his patent rights, and he sues the entrepreneur in federal court. The inventor's complaint alleges just enough facts of the entrepreneur's misconduct to support a possible claim for relief. The entrepreneur files a timely motion to dismiss, alleging that the inventor has failed to state a claim on which relief can be granted. The court grants the motion to dismiss Should the court have granted the motion to dismiss? (a) No, because the inventor's complaint contained just enough facts to support a possible claim for relief; (b) No, because the inventor held the patent on which the cause of action was based; (c) Yes, because the inventor's complaint contained just enough facts to support a possible claim for relief; (d) Yes, because the entrepreneur filed a timely motion to dismiss.

C

At a party for coworkers at Defendant's home, Victim accused Defendant of making advances toward his wife. Victim and his wife left the party. Defendant saw Victim and struck him on the head with a soft-drink bottle. Victim fell into a coma and died two weeks after the incident. This jurisdiction defines aggravated assault as an assault with any weapon or dangerous instrument and punishes it as a felony. It defines murder as the unlawful killing of a person with malice aforethought or in the course of an independent felony. Defendant may be found guilty of murder (a) only if the jury finds the Defendant intended to kill the victim; (b) only if the jury finds that Defendant did not act in a rage provoked by Victim's accusation; (c) if the jury finds that Defendant intended to kill or to inflict serious bodily injury; (d) if the jury finds that the killing occurred in the course of an aggravated assault

C

Frustrated with his marriage, a wealthy husband decides to seek a divorce. The husband and wife reside in State A. The husband hires an attorney, and after a painful and very expensive process, the husband and wife obtain a divorce decree in State court, in State A. Several years later, the former husband, who still resides in State A, believes that due to a substantial decrease in his income, he can no longer afford to pay the alimony he has been paying. He also believes he is entitled to a return of the portion of the alimony he has been paid to his former wife over the past several years. The former wife now lives in State B, and the former husband files a suit in federal court in State B seeking $80,000 in damages stemming from the initial alimony decree. The federal court should: (a) Hear the case because the parties are diverse and the amount in controversy is satisfied; (b) Hear the case because the federal court has jurisdiction over cases involving alimony; (c) Decline jurisdiction to hear the case because the case involves an alimony decree; (d) Decline jurisdiction to hear the case because both parties were domiciled in State A at the time of the divorce decree, which divested the federal court of diversity jurisdiction over the alimony claim.

C

One afternoon, a driver was driving his old beat-up truck along the highway when a police officer pulled him over. When the officer came up to the driver's side window of the car, the driver asked him, "What did I do wrong?" The officer replied, "Nothing, I just picked your car at random so that I could check your license and registration. May I see them?" The driver handed the officer his license and reached over to the glove compartment to get his registration. As the officer watched the driver do this, he noticed what appeared to be a handle of a gun protruding from under the passenger side of front seat. The gun turned out to be an illegal sawed-off shotgun. The officer arrested the driver on weapons charges. At trial, the driver's attorney moved to suppress the gun seized by the officer. The motion to suppress will most likely be (A) denied, because the handle of the shotgun was in plain view. (B) denied, because the seizure of the gun resulted from a lawful investigatory stop. (C) granted, because, as the fruit of an illegal search, the gun should be excluded. (D) granted, because the officer did not have probable cause to believe that the driver had or was engaged in criminal activity.

C

The United States caught a man in Afghanistan and alleged that he was an enemy combatant. The man countered that he was merely a humanitarian worker trying to help anyone in the area who needed food, water, and other basic necessities. The weapon he had on him was necessary for self-protection. The United States military houses the man at its facility in Guantanamo Bay, Cuba, where it houses other so-called enemy combatants. The man files a federal lawsuit, alleging that he is entitled to habeas corpus relief because he is being confined unconstitutionally. What is the most accurate statement about the validity or lack thereof of his claim to access to the courts? a) The man has no right to access the federal courts because he is a non-citizen; (b) The man has no right to access the federal courts because he has been declared an enemy combatant; (c)The man has a right to access the courts because he is entitled to at least some form of due process; (d) The man has a right to access the courts because international treaty obligations require it.

C

The attorney for a plaintiff in a civil action filed in federal district court served the defendant with the summons, the complaint, and 25 interrogatories asking questions about the defendant's contentions in the case. The interrogatories stated that they were to be answered within 30 days after service. The defendant is likely to succeed in obtaining a protective order on which of the following rounds? (a) Interrogatories are only proper to discover facts, not contentions. (b) A party can serve only 10 interrogatories, not 25. (c) Interrogatories may not be served until the parties have conferred to arrange for initial disclosures and prepare a discovery plan; (d) The interrogatories exceed the number permitted without permission from the court or an agreement between the parties.

C

While waiting in line to open an account with a bank, a customer read a poster on the bank's wall that said, "New Customers! $25 for 5 MINUTES. If you stand in line for more than five minutes, we will pay you $25! We like happy customers!" The customer started timing his wait and just as five minutes was about to pass, the bank manager tore the poster down and announced, "The $25 stand-in-line promotion is over!" The customer waited in line for 10 more minutes before being served. Does the customer have a claim against the bank for $25? (a) No, because the bank withdrew its offer before the customer completed the requested performance. (b) No, because the bank's statement was a nonbinding gift promise. (c) Yes, because the bank could not revoke its offer once the customer had commenced performance. (d) Yes, because the customer's presence in line served as notice to thee bank that he had accepted.

C

A city is concerned about the proliferation of adult businesses in its borders. City officials are upset about this, because they believe it could lead to the creation of a red-light district that would increase crime and decrease property values. City officials thus pass a zoning ordinance that provides that "no adult business can be located within 1,000 feet of any other existing adult business, any residence, any school, any daycare, or any other location in which children are likely to be present." An adult business owner hires a land-use planning expert who determines that there are only five locations currently existing in which adult businesses can locate. The owner decides to file a First Amendment lawsuit. What is the likely result? (A) For the owner, because the city did not leave open ample, alternative means of communication; (B) For the owner, because the city officials obviously acted out of a distaste for adult expression; (C)For the city, because the city's ordinance furthered the city's substantial interest in combatting adverse secondary effects associated with adult businesses; (D) For the city, because the adult businesses may have dabbled into obscenity.

C;

John Horofsky, a public school student, decides to run for elective office at his school. He knows that he is not the most popular kid at school and is not a star athlete. He decides that he needs to have a very catchy slogan to capture his fellow students' imagination. He decides to pass out pamphlets with the slogan "Horofsky for Student Body President - He Doesn't Take Shit from Limp School Officials." School officials object to this campaign activity, believing that it is not proper for the school environment. Horofsky manages to deliver the pamphlets. They do not cause any fights at school, though they do create a lot of conversation in the student hallways. The school officials warn Horofsky that if he distributes any more of his campaign materials, he will be removed from the political race and may face further disciplinary action. If Horofsky sues the school officials for violating his First Amendment free-speech rights, what is the likely result? (A) For Horofsky, because the campaign materials did not cause a disruption in school; (B) For Horofsky, because he engaged in political speech and the school officials imposed a prior restraint on his expression; (C)For the school officials, because his campaign materials contained profanity and are vulgar; (D) For the school officials, because school officials have absolute authority over students.

C; no 1A protection for public school student speech that is vulgar or lewd -Fraser test

A baker, who is a sole proprietor, enters into a contract with a grocer, who is the owner and sole proprietor of a small grocery store, to deliver fresh baked goods to the store by 6 A.M. every day except Sundays. The baker lives in the eastern district of State A and operates the bakery in that same district. The grocer lives in the western district of State B and operates the grocery store in the western district of State B. The contract negotiations and contract signing took place in the western district of State B. The baker meets his contractual obligations, but the grocer fails to pay the baker as required by the contract. The contract does not include a forum selection clause. The baker wants to file a lawsuit based on breach of contract. Where would venue be proper? (a) Venue would be proper only in the western district of State B. (b) Venue would be proper only in the southern district of State B. (c) Venue would be proper in State A because the baker lives in State A. (d) Venue would be proper in the southern or western district of State B.

D

A driver and a trucker are involved in an accident on a state road. The driver sues the trucker in federal court, alleging that the trucker's negligence caused the accident. The driver seeks $80,000 in damages. The case proceeds to a jury trial, during which the court makes an error during voir dire that seriously affects the trucker's substantial rights and causes an unfair prejudicial impact on the jury's deliberations. However, the trucker fails to object at the time of the error, and the court and driver are unaware of the error. The jury returns a verdict for the driver. The trucker hires a new attorney who, in reviewing the proceedings, notices the error. May the trucker raise the issue concerning the error on appeal? (a) No, because the trucker failed to preserve the issue; (b) No, because the trucker hired a new attorney who did not work on the case during the trial. (c) Yes, because the losing party has a fundamental right to select one unpreserved issue to raise on appeal. (d) Yes, because the error constituted plain error.

D

A driver from State A was operating a 1998 Camaro sports car on a highway in State B and was involved in a crash with a motorcyclist from State B. The motorcyclist was killed in the crash. The executor of the motorcyclist's estate is domiciled in State A. The driver believed that the accident was the fault of the motorcyclist's stunt-riding. Thus, the driver sued for negligence in federal court in State B, seeking $100,000 in damages from the motorcyclist's estate. Does the federal court have original jurisdiction over the case? (a) No, because the opposing parties are not citizens of different states. (b) No, because negligence is not a case or controversy arising under federal law. (c) Yes, because the Commerce Clause gives federal courts original jurisdiction over negligence claims that arise from transportation over highways within the United States. (d) Yes, because the opposing parties are citizens of different states, and the amount in controversy is $100,000.

D

A farmer was tired of living in the country. He placed the following announcement in the local paper. "I am interested in selling my farm for a reasonable price. First come, first served. Come on down." When a local real estate investor saw the ad, he could not believe his luck. He got into his car and raced to the property. As he stepped out of his car, the investor was greeted by the farmer, who informed him that he was the first to come in response to the ad. The investor declared his desire to purchase the property, but the farmer told him that he had decided not to sell the land. The investor filed an action seeking specific performance, in order to compel the farmer to sell him the property. Will the investor be successful? (A) Yes, because the real estate investor was the first person to respond to the announcement. (B) Yes, because the real estate investor communicated his desire to purchase the property. (C) No, because the announcement did not contain identifiable terms. (D) No, because the investor did not take possession of the property nor did he make improvements to the property.

D

A fire that started in the defendant's warehouse spread to the plaintiff's adjacent warehouse. The defendant did not intentionally start the fire, and the plaintiff can produce no evidence as to how the fire started. However, the defendant had failed to install a sprinkler system, which was required by a criminal statute. The plaintiff can produce evidence that had the sprinkler system been installed, it could have extinguished the fire before it spread. In an action by the plaintiff against the defendant to recover for the fire damage, is it possible for the plaintiff to prevail? (A) No, because the statute provides only for criminal penalties; (B) No, because there is no evidence that the defendant negligently caused the fire to start; (C) Yes, because a landowner is strictly liable for harm to others caused by the spread of fire from his premise; (D) Yes, because the plaintiff was harmed as a result of the defendant's violation of a statute that was meant to protect against this type of occurrence.

D

A local newspaper reports on the criminal trial of an armed robbery that led to the murder of a convenience store employee. Three men were convicted of both armed robbery and murder, including James Johannson, who drove the getaway car and participated in the robbery. However, Johannson did not shoot the clerk. However, the newspaper reported incorrectly that "Johannson was the triggerman." Johannson files a defamation lawsuit against the newspaper. If the newspaper prevails in the defamation lawsuit, what is the likely reason why? (a) Jurors don't like convicted criminals; (b) rhetorical hyperbole; (c) the substantial truth doctrine; (d) the libel-proof plaintiff doctrine.

D

A manufacturer sued a buyer in federal court for failing to make timely payments under the parties' sale contract. The case was tried to the court solely on documentary evidence. Immediately after the close of the evidence, the judge announced from the bench, "Judgment shall be entered for the manufacturer," and judgment was so entered. The buyer has appealed the judgment. What is the buyer's best argument for persuading the appellate court to reverse the judgment? (a) the judgment is clearly erroneous because it was based solely on documentary evidence; (b) the manufacturer was required to file proposed findings and conclusions before the trial court ruled; C) the trial court erred because it announced the judgment without giving the parties an opportunity to submit proposed findings and conclusions; (d) the trial court erred by not providing findings and conclusions.

D

A plaintiff corporation incorporated in State A with its principal place of business therein entered into a contract with the defendant under which the defendant agreed to manufacture and sell equipment to the plaintiff. The plaintiff plans to sue the defendant for $1 million because the equipment does not conform to the contract specifications and does not operate properly. The defendant is a corporation that is also incorporated in State A but has its principal place of business in State B. Would a federal district court have subject matter jurisdiction over the plaintiff's action? (a) Yes, because the two corporations are citizens of different states based on their principal place of business; (b) Yes, because the transaction involves interstate commerce; (c) Yes, because both parties have sufficient contacts with State A; (d) No, because the two corporations are both citizens of State A based on their place of incorporation.

D

A plaintiff files a complaint alleging that the defendant failed to meet contractual obligations. The defendant's attorney files several baseless motions in bad faith. The plaintiff properly files a motion for sanctions against the defendant. After hearing the matter, the court imposes sanctions against the defendant, including an order that the defendant pay a penalty into court. Was the court's order permitted? (a) No, because the court may not impose sanctions against a party prior to trial. (b) No, because a court may not impose a sanction requiring a party to pay a penalty into court. (c) Yes, because when a court finds that sanctions are appropriate, the court must require the sanctioned party to pay a penalty into court. (d) Yes, because a court may impose a sanction requiring a party to pay a penalty into court.

D

A public school student wishes to distribute copies of an underground student newspaper at his school. One of the articles talks about censorship in school. The principal, believing the content to be borderline offensive, prohibits the student from distributing his work at school. He tells the student that he can distribute the paper about a block from school grounds at a local store. If the student's parents file a lawsuit on behalf of their minor son, what is the likely result? (a) For the principal, because he believed the newspaper was offensive; (b) For the principal, because there is a more deferential standard for school-sponsored student expression; (c)For the student, because underground student newspapers are absolutely protected under the First Amendment; (d) For the student, because there was no indication that the underground newspaper would disrupt school activities.

D

A regulation prevents persons with a drug arrest and conviction and/or those undergoing current treatment for drug abuse from working in certain state run facilities for the safety of the general public. One employee of the department of treasury brought an action claiming a denial of equal protection and due process by taking away his right to a job. It is claimed that the prohibition is necessary to protect the public from potential problems that could arise. Which test of constitutional validity would be likely applied to this regulation? (a) Strict scrutiny, since drug users are a special class of persons often discriminated against by the public. (b) Intermediate scrutiny, because, like gender, this can be a sensitive area where discrimination is common. (c)The rational basis test, because discrimination is generally favored with drug users or even those getting treatment. (d) The rational basis test, because the regulation need only be related to a legitimate state interest to be valid

D

A schizophrenic patient who was institutionalized in a psychiatric facility pushed a nurse down a stairwell at the facility. The nurse, a paid employee of the facility who was trained to care for schizophrenic patients, was injured. The patient is an indigent whose care is paid for by the government. The jurisdiction generally follows the rule that a person with a mental deficiency is held to the standard of a reasonable person. In a negligence action brought by the nurse against the patient, the patient's lawyer will argue that the patient should not be held responsible for the nurse's injury. Which of the following facts will be LEAST helpful to the patient's lawyer's argument? (A) The nurse was a professional caregiver; (B) The nurse was trained to care for patients with schizophrenia; (C) At the time she pushed the nurse, the patient thought she was being attacked by an elephant; (D) The patient is an indigent whose care is paid for by the government.

D

A trucker who works for a trucking company is traveling at a high rate of speed during a snowstorm while working for the company. The trucker loses control of the truck and is involved in an accident with a driver. The company prepares a report to address a threat of imminent litigation with the driver regarding the accident. Two weeks after the report is completed, the driver files a claim in federal court against the company. The driver seeks $100,000 from the company for damages sustained in the accident. During discovery, the company refuses to disclose the report, claiming that it is not discoverable. Is the company correct in its assertion that the report is not admissible? (a) No, because it is relevant information pertaining to the accident. (b) No, because the lawsuit was not pending at the time the report was completed. (c) Yes, because the company need only disclose the report upon motion by the driver. (d) Yes, because the report was prepared to address a threat of imminent litigation.

D

An environmentally friendly company wanted to plant some eucalyptus trees around the perimeter of the company's property. The company contacted a nursery to see about purchasing the trees. The nursery estimated that the company needed 500 trees, and the nursery offered the trees to the company at $5 per tree, delivery included. The nursery indicated that they would keep their offer open until August 1. On July 15, the company wrote to the nursery asking if the nursery would sell 400 trees to them under the same terms. The nursery did not respond. On July 20, the company sent a telegram to the nursery indicating that they wanted to purchase 500 trees at $5 each, delivery to the company's property to be included. Which of these is the proper result? (A) The company's July 15 letter constituted the company's substantial acceptance of the nursery's offer. (B) The company's July 15 letter constituted a counteroffer that was not accepted by the nursery. (C) The company's July 20 telegram was not an effective acceptance, since the nursery did not indicate they would accept an offer contained in a telegram. (D) The company's July 20 telegram formed a valid contract with the nursery.

D

An inventor holds a patent for a device that increases fuel economy in gasoline-fueled combustion engines. An opportunist learns of the inventor's device and begins manufacturing and selling an identical device, marketing it as the opportunist's own invention. The inventor sues the opportunist in federal court on a claim of patent infringement. The court enters a final judgment in favor of the inventor. The opportunist appeals the judgment, arguing that the court incorrectly applied the law. What standard will the appellate court apply when reviewing the opportunist's appellate claim? (a) abuse of discretion; (b) clearly erroneous; (c) preponderance of the evidence; (d) de novo

D

An underworld informer advised a police investigator that his neighbor was running an illegal bookmaking operation in his apartment, and that the informer had placed bets with the neighbor at this location. The officer obtained a search warrant, based on his affidavit reciting the foregoing facts, and further stating that the underworld informer was a person who had given him accurate information in previous cases, but whose identity could not be jeopardized because of other pending criminal investigations. Armed with the search warrant, police officers went to the neighbor's apartment. They entered when the neighbor opened the door and searched the apartment. They seized various wagering slips and bookmaking apparatus (described in the search warrant) and placed the neighbor under arrest for illegal gambling. Prior to trial, the neighbor challenges the validity of the search warrant. Was the search valid? (A) no, because it was based on hearsay information; (B) no, because the officer failed to disclose the name of the informer; (C) yes, because the identity of the informer is never required; (D) yes, because the affidavit accompanying it is sufficiently detailed to allow a determination of probable cause

D

Billy Bowers is a rugged power forward who plays professional basketball. He is known for playing a particularly aggressive brand of basketball. In one game, Bowers commits a hard foul, which caused the player he fouled to suffer an injury. James Johnson, a local sports columnist, writes a column in which he writes: "Billy Bowers is a thug who should be suspended by the commissioner. We don't need thugs like this in the game." Bowers, who has no criminal record, sues Johnson for defamation. What is the likely result? (a) For Bowers, because he did not mean to cause an injury to the opposing player; (b) for Bowers, because he has no criminal record and the word "thug" connotes unlawful activity; (c) for Johnson, because Bowers plays thuggish basketball; (d) for Johnson, because his language was a form of rhetorical hyperbole.1.

D

Several studies have concluded that the likelihood of being an adult cigarette smoker is greater when a smoking habit is started during adolescence. In response, the U.S. Congress passed a bill prohibiting the placement of any billboard advertising cigarettes or any other tobacco products within 300 feet of any public elementary school, middle school, or high school. Which of the following would provide the best ground for challenging the constitutionality of the federal law? (a) The commerce clause; (b) The due process clause of the 5th Amendment; (c)The equal protection clause of the 5th amendment; (d) The First Amendment

D

Suffering from terminal and painful cancer, Willa persuaded Harold, her husband, to kill her to end her misery. As they reminisced about their life together and reaffirmed their love for each other, Harold tried to discourage Willa from giving up. Willa insisted, however, and finally Harold held a gun to her head and killed her. The most serious degree of criminal homicide of which Harold can be legally convicted is ... (a) no degree of criminal homicide; (b) involuntary manslaughter; (c) voluntary manslaughter; (d) murder

D

The police are called into a public park because of various noise complaints. It turns out that a rock music band was playing music at very high decibel levels. The police determined that this violated a local noise ordinance. The police started to approach the stage to stop the concert. As the police approached the stage, one of the band members yelled to the group of spectators: "**** the Police. Resist the Police. Stop Tyranny." Immediately, several members in the audience hurled bottles at the police officers. The officers arrested the band member and charged him with inciting a riot. If the man asserts a First Amendment-based defense, what is the most likely result? (A) For the man, because he engaged in protected political speech in a free society; (B) For the man, because the man did not specifically call for people to hurl bottles at the police officers; (C) For the government, because the band member engaged in fighting words; (D) For the government, because the band member incited imminent lawless action.

D

A rap group released a track that featured loads of profanity, misogyny, and violent imagery. One song featured graphic depictions of sexual activity, violence towards women, and a constant refrain about engaging in illegal activity. A local prosecutor believes that this material is harming the city's youth and contributing to youthful delinquency. The prosecutor files a declaratory judgment action, asking a judge to have the material declared legally obscene. Assume for purposes of the question that this is a cognizable procedural mechanism in this jurisdiction. Which statement is NOT accurate with regard to determining whether the material crosses the line from protected expression to unprotected obscenity? (A) The group's songs contains unhealthy and shameful interests in sex; (B) The group's song contains graphic depictions of explicit sexual activity; (C)The material has serious artistic value, according to a music critic; (D) The material is quite offensive to the judge's personal predilections.

D; Miller test prongs

A man lent his friend his sterling silver, antique punch bowl for the friend's annual Christmas party. The party was a rousing success. As the man was leaving the party, he told the friend that he needed the bowl back in two days or his wife would have a fit. The friend ended up keeping the man's punch bowl for two weeks. On the date in question a small plane lost power and crashed into the friend's house, causing massive damage and starting a fire. The punch bowl was destroyed in the crash. If the man sues the friend based on negligence, what is the likely outcome? (A) Finding for the man, because the friend breached a verbal contract by keeping the punch bowl longer than authorized. (B) Finding for the man, because if not for the delay in the return of the punch bowl, it would not have been destroyed. (C) Finding for the friend, because he cared for the punch bowl properly. (D) Finding for the friend, because a plane crashing into his house was an unforeseeable risk.

D; The plane crashing into the house is a superseding event that is outside of the scope. If this was a suit for conversion not negligence, this would be different.

A daughter and her boyfriend have been dating for several years. The daughter's father objects to the relationship and tells several of his friends and acquaintances that if the boyfriend doesn't leave his daughter alone, the father will "tear the boyfriend limb from limb." The father never threatens the boyfriend to his face. The father is a retired professional wrestler who was forced into retirement for using excessive force in the ring. The boyfriend sues the father for assault. Should the boyfriend prevail in his assault case? (A) Yes, because the father has the apparent ability to carry out his threat. (B) Yes, because the father has made the threat in the presence of multiple individuals. (C) No, because the boyfriend has incurred no damages as a result of the alleged assault. (D) No, because a threat of future violence generally does not constitute an assault.

D; generally words alone are not enough to rise to the level of assault

. As a joke, the defendant knocked on the plaintiff's door wearing a police officer's uniform that he had rented from a costume shop. When the plaintiff came to the door, the defendant told her that her husband had just been killed in a highway accident, and that she would have to come with him to claim the body. The plaintiff, who recognized the defendant and knew that he was not a police officer, slammed the door in his face and told him to leave her alone. She was outraged at his attempt to play such a joke on her, but she sustained no physical or mental injury. If the plaintiff asserts a claim against the defendant for IIED, the court should find for: (A) the plaintiff, because the defendant's conduct exceeded all bounds normally tolerated by decent society; (B) the plaintiff, because the defendant's conduct was calculated to cause severe mental suffering; (C) the defendant, because his intention was merely to play a joke on the plaintiff; (D) the defendant, because the plaintiff sustained no physical or mental injury as a result of the defendant's conduct

D


Related study sets

CH. 01: Introduction to Networking

View Set

GEO 103 (Dr. J.) - Test #7 Study Guide

View Set

Master Algebra 2 (except 11.4 and half of chp. 7)

View Set

CH 13 Financial Statements and Closing Procedures

View Set

Chapter 15-Monitoring & Auditing AIS

View Set

A2: Types of Private Insurers Quiz

View Set